You are on page 1of 113

*If u find they said in front of the answer >>>> I meant TAGOG members.

1) Child
a.
b.
c.
d.

present with stiffing neck, fever, headach. You suspect meningitis what is your initial TTT should be:
Tobramycin
Levoflaxicine
Peneciline(ampicillin)
Doxycycline
I remember there is no vancomycine, ceftriaxone or dexamethasone on the choices.
-The correct answer is c.

AGE

Causative organism

Treatment

< 1 MONTH

GBS, E coli

Ampicillin + cefotaxime or
gentamicin

1-3 MONTHS

S.pneumonia,
H.infulenza
Meningocci

Vancomycin + cefotaxime or
cefriaxone

3 MONTHS ADULT

Pneumoccoci,
meningococci

Vancomycin + cefotaxime or
ceftiaxone

Pneumococi,
meningococci
Gram ve bacilli

Ampicillin + vancomycin +
ceotaxime or ceftiaxone

>60
YEAR\acoholism

2) Link the ttt with organism:


a. Shegella metronidazole
b. Salmonella erythromycin
c. Combylobacter amoxacilline
d. Gardia I foregut ( but I remember it is antibiotic)

-All of the above answers are wrong.?


3) Female presented with vaginal discharge, itching, and on microscope showed mycoleous cells and spores. This
medical condition is most likely to be associated with:
a. TB
b. Diabetes
c. Rheumatoid Arthritis
The correct answer is b
Vaginal thrush is a common infection caused by a yeast called Candida albicans. Vulvovaginal candidiasis is usually
secondary to overgrowth of normal flora Candida species in the vagina. Conditions that interrupt the balance of
normal vaginal flora include: antibiotic use, oral contraceptives, contraceptive devices, high estrogen levels, and
immunocompromised states such as diabetes mellitus and HIV. Women are prone to vaginal thrush between puberty
and the menopause because, under the influence of the hormone estrogen, the cells lining the vagina produce a
sugar and yeasts which Candida albicans are attracted to. That is why thrush is rare before puberty.
4) Trichomoniasis is classically have:
a. Clue cells
b. Greenish frothy discharge
The correct answer is b
Trichomoniasis is caused by the flagellated protozoan Trichomonas vaginalis; it's the most common curable
sexually transmitted disease in the world. It usually presents with frothy yellow-green vaginal discharge, strongunpleasant odor, pain during urination and sexual intercourse.
5) Obsessive neurosis:
a. Treatment is east
b. Clomipramine doesnt not work
c. Mostly associated with severe depression
d. Can be cured spontaneously
the correct aswer is c
6) Female had history of severe depression, many episodes, she got her remission for three months with Paroxitine
( SSRIs) .. now she is pregnant .. your advise
a. Stop SSRi's because it cause fetal malformation
b. Stop SSRi's because it cause premature labor
c. Continue and monitor her depression
d. Stop SSRIs

The correct answer is c or a Some women, will choose to remain on paroxetine (both prior to conception and
during pregnancy) because it has been effective for them. We do not recommend that these women switch to
another drug.So the answer is C.
7) Hallucinations and Paranoia:
a.
b.
c.
d.

SCZ
Mood
Mania
Phobia

The correct answer is a


8) One of the following decrease chance of colon cancer :
a.
b.
c.
d.

Zinc
Vit. E
Vit C
Folic acid

????????
Both C, E, Beta carotene all are Antioxidants, they are correct but C is more likely. A big dose of vitamin C fights
the big "C." some others: Fiber ,Vitamin C, Calcium, Vitamin E, Selenium. Zinc also .the answer is d. A metaanalysis of eight controlled trials found no convincing evidence that antioxidant supplements had a significant
beneficial effect on primary or secondary prevention of colorectal adenomas
9) patient complains of "can't breathe air in one nostril "; on examination showed edematoud mucosa structure, best to
give initially :
a. Corticosteroids
b. Decongestants
c. Alfa-adrenergic blockers
The correct answer is a
10)
Female presented with complain of neck pain and occipital headache , no other symptoms , on X-ray has
cervical spine osteophyes and narrow disks :
a. cervical sponylosis ?
??????

11)
Patient complaining of pain at night when he elevated his arm, tingeling on lateral arm side and lateral three
fingers , Dx
a. brachial plexus neuropathy
b. shoulder impengment syndrom
c. brachial artery thrombophebitis
d. thoracic outlet problem
the correct answer is d
Branchial plexus neuropathy is characterized by acute onset of intense pain in the shoulder or arm followed
shortly by focal muscle weakness.
Impingement syndrome, swimmer's shoulder, and thrower's shoulder, is a clinical syndrome which occurs when
the tendons of the rotator cuff muscles become irritated and inflamed. This can result in pain, weakness and loss
of movement at the shoulder. The pain is often worsened by shoulder overhead movement and may occur at
night, especially if the patient is lying on the affected shoulder.
Thoracic outlet syndrome: numbness and tingling in the fingers; pain in the neck, shoulder, and arm; weakness of
the arm and dropping things from the hand; worsening of the symptoms when elevating the arm to do such things as
comb or blow dry one's hair or drive a car; and coldness and color changes in the hand. The symptoms are often
worse at night or when using the arm for work or other activities.
12)
male came to you complaining of sudden progressive decreasing in vision of left eye over last two/three days,
also pain on the same eye, on fundoscopy optic disk swelling was sees , Dx :
a. central retinal artery occlusion
b. central retinal vein occlusion
c. optic neuritis
d. macular degeneration
they said the correct answer is c

Ask about symptoms of temporal arteritis in the older population. Patients complain of sudden, painless,
nonprogressive vision loss in one eye. History of headaches, jaw claudication, scalp tenderness, proximal muscle and
joint aches, anorexia, weight loss, or fever may be elicited.

Some patients may reveal a history of amaurosis fugax involving transient loss of vision lasting seconds to minutes
but which may last up to 2 hours. The vision usually returns to baseline after an episode of amaurosis fugax.

Central retinal Artery occlusion: presenting complaint is an acute sudden painless unilateral loss of vision. Shows a
classic cherry red spot

Central retinal vein occlusion: usually sudden painless variable visual loss; the fundus may show retinal hemorrhages,
dilated tortuous retinal veins, cotton-wool spots, macular edema, and optic disc edema.

Optic neuritis: Major symptoms are sudden loss of vision (partial or complete), or sudden blurred or "foggy or washed
out" vision, and pain on movement of the affected eye. Many patients with optic neuritis may lose some of their color
vision in the affected eye (especially red). The optic disk becomes swollen

Macular degeneration is a leading cause of permanent painless irreversible vision loss in the elderly.

Retinal Detachment; Symptoms are decreased peripheral or central vision, often described as a curtain or dark cloud
coming across the field of vision. Associated symptoms can include painless vision disturbances, including flashing lights
and excessive floaters.
13)
70 years old with progressive demntia , no personality changes , neurological examination was normal but there
is visuodeficit , on brain CT shower cortex atrophy and ventricular dilatations :
a. multi micro infract dementia
b. alzehimer demenita
c. parkinsonism dementia
the correct answer is b
alzehimer dementia :
most common cause of dementia. age and family history are risk factors for AD. Etiology unknown but toxic bamyloid deposit in brain. Present with amnesia for newly acquired information is usually the first presentation,
followed by language deficit , acaluia, depression, agitation and finally apraxia(inability to perform skilled
movement). Diagnosis by exclusion that can be definitive diagnosis only on autopsy: suggested by clinical feature
and by progressive cognitive course without substantial motor impairment. MRI & CT may show atrophy , venticule
enlargement and can rule out other causes. On brain microscopy amyloid plaques and neurofibrially tangle. Death
usually occurring secondary to aspiration pneumonia . treatment by supportive therapy for Pt. and family , and
cholinesterase inhibitor .

multi mico infarct dementia ( vascular demensia )


dementia associated with history of strok. Criteria for vascular dementia include presence of dementia and 2 or
more of the following:
1- focal neurological signs
2- symptoms onset that was abrupt , stepwise, or related to strok
3- brain imaging showing evidence of fold infarction or extensive deep white matter changes secondary to chronic
ischemia.
Alzheimer (Dx by exclusion. Its associated with progressive memory loss, decreased cognition function , & enlarged
ventricles with cortical atrophy)

multi infarct dementia (NOT progressive & it has focal neurological abnormality)

multiple sclerosis (recurrent relapsed & complete remission. Its associated with demylenation of gray-matter)

14)

Young adult presented with painless penile ulcer rolled edges .. what next to do :
a. CBC
b. Darkfeild microscopy
c. Culturing
The correct answer is b

15)

Syphilis also known as "great imitator" is a sexually transmitted disease caused by the spirochete bacteria
Treponema pallidum. Classically presents as single painless non-itchy skin ulceration with sharp borders. T
pallidum is too small to be seen under the light microscope. So use darkfield microscopy when sores are
present. Blood tests can confirm the presence of antibodies. The antibodies remain in your body for years, so
the test can be used to determine a current or past infection.
which prevent or decrease incidence of getting post herpetic neuralgia
a. Amitriptylin
b. Acyclovir
c. varicella vaccination
d. valacyclovir
The correct answer is c
choose the best it will be varicella vaccination, if the rash start give valacyclovir it better than acyclovir

16)
mother gave bitrh of baby with cleft lip and palate, she want to get pergnant again what is the percentage of
recurrence
a. 1%
b. 4%
c. 15%
The correct answer is b
17)
4 years old girl presented with her parents to er with sore throat and seroangious vaginal discharge with no pain
what is most propable cause,
a. Candida
b. Foreign body
c. Chlamedia
d. Gonococci
e. streptococcus
The correcr answer is e
Pediatric vaginal discharge:
1- infectious vulvovaginitis: present with malodorous , yellow green, most common caused by group A
streptococcus. ( may be present with sexual abus "STDs" )
2foreign body
3-candidal infection: may associated with diabetes measure glucose.
4-Sarcoma botryoids (rhabdomyosarcoma ): malignant lesion appearance of "bunches of graps" within
vagina
18)patient coplaining of pain along median nerve ditribution , And positive tinel sign treatment include casting of both
hand in what position
a. Dorsiflexion
b. plantar flexion
c. extention
d. Dduction
The correct answer is c
19)
dermatomyosistis what is true
a. distal muscle weakness
b. Underlying malignancy
c. Generalized ??? Skin rash

The correct answer is c


Polymositis & dermatomyositis :
Polymyositis : progressive , systemic tissue disease characterized by immune-mediate striated muscle inflammation,
present with symmetric progressive proximal muscle weaknes and pain .
DERMATOMYOSITIS : present polymyositis plus cutaneous involvement, heiotrop rash (violaceous periorbital rash) ,
shawl sign ( rash involving the shoulder, upper chest and back ) , Gottron"s papule ( popular rash with scale ).
-both : increase serum CK and anti-Jo antibodies . muscle biopsy uscle fiber and inflammation.
20)

pt taking bupropion to quit smoking what is SE


a. Arrythmia
b. Seizure
c. xerostomia
d. Headache
The correct answer is b

the answer is d.

Frequencies, when reported, reflect highest incidence reported with sustained release product.
>10%:
Cardiovascular: Tachycardia (11%)
Central nervous system: Headache (25% to 34%), insomnia (11% to 20%), dizziness (6% to 11%)
Gastrointestinal: Xerostomia (17% to 26%), weight loss (14% to 23%), nausea (1% to 18%)
Respiratory: Pharyngitis (3% to 13%)
1% to 10%:
Cardiovascular: Palpitation (2% to 6%), arrhythmias (5%), chest pain (3% to 4%), hypertension (2% to 4%; may be
severe), flushing (1% to 4%), hypotension (3%)

Central nervous system: Agitation (2% to 9%), confusion (8%), anxiety (5% to 7%), hostility (6%), nervousness (3% to
5%), sleep disturbance (4%), sensory disturbance (4%), migraine (1% to 4%), abnormal dreams (3%), irritability (2%
to 3%), somnolence (2% to 3%), pain (2% to 3%), memory decreased (3%), fever (1% to 2%), CNS stimulation (1%
to 2%), depression
Dermatologic: Rash (1% to 5%), pruritus (2% to 4%), urticaria (1% to 2%)
Endocrine & metabolic: Menstrual complaints (2% to 5%), hot flashes (1% to 3%), libido decreased (3%)
Gastrointestinal: Constipation (5% to 10%), abdominal pain (2% to 9%), diarrhea (5% to 7%), flatulence (6%), anorexia
(3% to 5%), appetite increased (4%), taste perversion (2% to 4%), vomiting (2% to 4%), dyspepsia (3%), dysphagia
(2%)
Genitourinary: Polyuria (2% to 5%), urinary urgency (2%), vaginal hemorrhage (2%), UTI (1%)
Neuromuscular & skeletal: Tremor (3% to 6%), myalgia (2% to 6%), weakness (2% to 4%), arthralgia (1% to 4%),
arthritis (2%), akathisia (2%), paresthesia (1% to 2%), twitching (1% to 2%), neck pain
Ocular: Blurred vision (2% to 3%), amblyopia (2%)
Otic: Tinnitus (3% to 6%), auditory disturbance (5%)
Respiratory: Upper respiratory infection (9%), cough increased (1% to 4%), sinusitis (1% to 5%)
Miscellaneous: Infection (8% to 9%), diaphoresis (5% to 6%), allergic reaction (including anaphylaxis, pruritus, urticaria)
<1% (Limited to important or life-threatening): Accommodation abnormality, aggression, akinesia, alopecia, amnesia,
anaphylactic shock, anemia, angioedema, aphasia, ataxia, atrioventricular block, bronchospasm, bruxism, colitis, coma,
coordination abnormal, cystitis, deafness, delayed hypersensitivity, delirium, delusions, depersonalization, derealization,
diplopia, dysarthria, dyskinesia, dyspareunia, dysphoria, dystonia, dysuria, edema, EEG abnormality, emotional lability,
erythema multiforme, esophagitis, euphoria, exfoliative dermatitis, extrapyramidal syndrome, extrasystoles, facial
edema, gastric reflux, gastrointestinal hemorrhage, glossitis, glycosuria, gum hemorrhage, gynecomastia,
hallucinations, hepatic damage, hepatitis, hirsutism, hyper-/hypoglycemia, hyper-/hypokinesia, hypertonia,

hypoesthesia, hypomania, impotence, intestinal perforation, intraocular pressure increased, jaundice, , leukocytosis,
leukopenia, libido increased, liver function abnormal, lymphadenopathy, manic reaction, MI, muscle weakness,
musculoskeletal chest pain, mydriasis, myoclonus, neuralgia, neuropathy, painful erection, pancreatitis, pancytopenia,
paranoia, pneumonia, photosensitivity, postural hypotension, pulmonary embolism, rhabdomyolysis, salpingitis,
sciatica, seizures (dose-related), SIADH, stomach ulcer, Stevens-Johnson syndrome, stomatitis, stroke, suicidal ideation,
syncope, tardive dyskinesia, thrombocytopenia, tongue edema, urinary incontinence, urinary retention, vasodilation
21)
14 years old girl complaining of painless vaginal bleeding for 2-4 days every 3Weeks to 2 months ranging from
spotting to 2 packs per day; she had 2ry sexual ccc 1 year ago and had her menstruation since 6 months on clinical
examination she is normal sexual ccc, normal pelvic exam appropriate action
a. OCP can be used
b. You should ask for FSH and prolactin level
??????????????
22)

ttt of cyclical mastalgia


a. OCP, analgisc, NSAID, Fat reduction, and magnisuem
Mastalagia : painful breast tissue that can be cyclic and usually associated with hormonal change, often bilateral
. management : stop current hormonal therapy , reassurance, stop smoking, fat reduction, analgesic, NSAID,
OCP.

23)

4years old child what can he do


a. Copy square and triangle
b. Speak in sentences
b. ..
The correct answer is a

24)

baby can sit without support, walk by holding fourniture. Pincer grasp, pull to stand how old is he
a. 8 months
b. 10 months
c. 12 month
d. 18 month

The correct answer is b


25)
62 female with ve pap smear you should advice to repeat pap smear every:
A- 6m
b- 12m
c- 18m
d- no repeat
The correct answer is d
Screening pap smear:
1- starting at age 21 years or no more than 3 years after becoming sexually active.
2- women > 30 years who have three consecutive normal test screening ( 1 / 3yeasr).
screening should be discontinue for women > 60-70 years who have had 3 or more normal pap smear.

3-

26)
50 y with uncontrolled diabetes ,complain of black to brown nasal discharge. So diagnoses is
a- mycomyosis
b- aspirglosis
c-foreign body
d????????
The correct answer is a
- mycomyosis (fungal infection caused by Mycorales, affect nasal sinus & lungs, .
. characterized by black nasal discharge, Dx by biopsy).
27)
clonidin is decrease effect of
a- benzotropin (anticholinirgic for Parkinson. Not affected by Clonidin)
b-levo dopa (for Parkinson. Changed in the brain to Dopamen. Clonidin the effect of Levodopa through Unknown
mechanism)
c-rubstin?????
the correct
answer is b
Clonodin is 2 agonist used to TTT hypertension. 2 receptor in the brain cause of both COP & peripheral
resistant .

28)
6m baby with mild viral diarrhea , ttt by ORS as
a-100ml/kg for 4 hour then 50 ml/kg /day after
b-50>>>>>>>>>>>>>>>>.50>>>>>>>>>>
c-100>>>>>>>>>>>>>>>>100>>>>>>>>
d-50>>>>>>>>>>>>>>>>>100>>>>>>
29)
25y female with bradicardia and palpitation. ECG normal except HR130 and apical pulse is 210 .past history of
full ttt ovarian teratoma, so your advice is
a- struma ovari should be consider
b-vagal stimulate should be done
c- referred to cardiology
The correct answer is a
30)
50y man with chronic psychosis and not complains for ttt .your advice
a- depot haloperidol or floxtin
b-oral lorasepam
c-oral buspiron
d-???????
The correct answer is a
31)
15y boy appear patch in rt lower leg these patch is clear center , red in peripheral, no fever no other complain
so diagnosis
a-contact dermatitis
b-tinea corpora
c- lyme disease
d-???????
The correct answer is b
-tinea corpora: worm fungal infection which transmitted by contact skin & has clinical picture as in the question .
32)
pt with heart disease complain of LL ischemia your advice
a-referred to cardiology
b-""""""""""""vascular surgery
c- start heparin
d-??????????/

?????????
33)
7y boy complain of limb. CT show avascular necrosis in epiphysis of femur your advice
a- surgical ttt
b-splint for 6m
c- physiotherapy
d????????
Q not complete: but with these MCQs , the correct answer is b
This case is "perth's diseas" : vascular necrosis of femoral head. 5- 10 years . usually self limiting with symptoms
lasting< 18 months. Present with painless limb , limited abduction and internal rotation.
Treatment: 1- observation if there is limited femoral head involvement or full ROM .
2- if extensive or if decrease
ROM, consider bracing, hip abduction with cast.
34)
pt with trachoma in eye . for prevention you should
a- water
b- """""""""""""""""+eradication of organism
c- mass ttt
d???????????
the correct answer is a
-trachoma in the eye is a bacterial infection caused by Chlamydia trachomatis which is transmitted by poor haygen &
contaminated H2O. TTT by antibiotic as erythromycin & Doxycycline. Surgery to prevent scar
35)
your advice to prevent plaque disease is
a-hand washing
b-rodent eradication
c-spry insect side
d-??????????
36)
pt with severe headache and decrease in visual acuity ,pupil is dilated, so ttt
a-pilocarpin drop and ophthalmology referred
b-ergotamine
c-NSID
d???????
the correct answer is a

- this is closure angle glaucoma which characterized by sudden severe headache, red eye, visual acuity, &
dilated pupil. Pilocarpin is parasympathomimmic which help in relive the pain
37)
main ttt of non inflammatory acne is
a-ritonic acid
b-clindmycin;
c-azalic acid
d-erythromycin
the correct answer is a
.
.

-clindamycine & erythromycin are treatment of inflammatory acne.


azalic acid : treatment of non-inflammatory & inflammatory acne.
ritonic acid : treatment of sever acne & non-inflammatory

38)
pt with scale in hair margin and nasal fold and behind ear with papule and irregular erythema so ttt is
a-nizoral cream
b- atovit
c- acyclovir
d-antibiotic
the correct answer is a ( this is seborrheic dermatitis )
39)

question about diarrhea and Yesinia bacteria


Yersiniosis : it is infectious disease caused by Yesinia bacteria. There are 3 types of Yesinia bacteria : 1Y.entercolitis , 2- Y.pseudotuberculosis, 3-Y. pestis. Y.enetercolitis cause bloody diarrhea, terminal ileitis and
mesenteric adenitis. Diagnosis by serological : rise in antibody titer . Treatment : usually self limiting and
tetracycline for sever infection.

40)
a- I
b-II
c-III
d-IV

paraplegia pt with ulcer in lower back 2+2 cm and lose of dermis and epidermis these ulcer in stage

the correct answer is b


-stage I : non-blanchable redness that NOT subside after relive of the pressure
-stage II : damage to epidermis & dermis but NOT deeper

-stageIII : subcutaneous tissue involvement


-stageIV : deeper than subcutaneous tissue as muscles & bones
41)

female complain of painless odorless and colorless vaginal discharge that appear after intercourse so ttt
a-give antibiotic
b-douche after intercourse
c- cervical cancer should be consider
d-may be due to chronic salpingitis
the correct answer is b

Color

Smell

Pain OR
Itching

Other

Normal
vaginal
discharge

Clear OR
milky

Odorless

NO

during
ovulation,
sex,
breastfeedin
g

Bacterial
vaginosis

White-gray
OR yellow

Fishy

Yes

during sex

Trichomonia
sis

Watery Or
yellow

Yes
especially
during
urination

Candida

Whitecheesy thick
sticky

Yes
especially
during sex

42)

UTI>14 day, most probably cause pylonphritis


a-,05%
b-,5%
c-5%
d-50%
Difficult Q , This Q referral to urologist.

43)

44)

pt with long history of UC on endoscopes see polyp and cancer lesion on left colon so ttt
a-ttt of anemia
b-left hemicolctomy
c-total colctomy
d- remove polyp
boy 3 day after flue symptom develop conjunctivitis with occipital and neck L.N enlarged so diagnosis is
a-adenoviruses
b-streptococcus
c-HSV
D??????????
the correct answer is a

45)

.
46)

child with asthma use betamethazone, most common side effect is


a-increase intraocular pressure
b-epilepsy
c-growth retardation
d-?????????????
the correct answer is c
sickling pt after acute attack , discharge on
a- penicillin
b-iron
c-vitamin
d???????????
the correct answer is a
prophylactic of pneumococcal infection by vaccination and oral penicillin

47)

6m with cough and wheezy chest .diagnosis is


a- asthma
b- broncholitis
c-pneumonia
d-F.B aspiration
The correct answer is b
- asthma : after 2 years old)
- broncholitis : before 2 years old)
-pneumonia : associated with crypitation
-F.B aspiration : sudden wheezing

48)
15y old with pilonidal sinuse so ttt
a-incision surgery
b- local antibiotic
c-daily clean
d-?????????
The correct answer is c
.
pilonidal sinus : sinus tract witch commonly contain hairs. Treatment , firstly conservative ttt in mild case remove
all hair, washing cleaning . if not relive: surgery

49)
Female pt 8 wks postpartum,not smoker diagnosed to have asthma,her asthma was not controlled she attended
ER 3 times last month,on B2 agonist and oral steroid,she came c/o wheezing and s.o.b mildly cyanosed using her
intercostal muscles,wheezy chest,BP:160/100 P:120 PO2:72 PEF:36,there is oedema in her foot up to the knee,the
most likely diagnosis:
a. COPD
b. pulmonary embolism
c. Acute asthma attack
d. Angioedema

The correct answer is a or c


50)

male pt developed corneal ulcer in his Rt eye after trauma what is the Mx:
a. Antibiotic and cycloplagia is mydrasis and refer to ophthalmology
b. topical steroid
the correct answer is a
Because infection is a common occurrence in corneal ulcers, your ophthalmologist will prescribe antibiotic
eyedrops. If the infection appears very large, you may need to use these drops as often as one drop an hour. -Oral
pain medications will be prescribed to control the pain. Pain can also be controlled with special eyedrops that keep
your pupil dilated (Anticholinergics such as atropine, hyoscyamine, and scopolamine)

51)
female pt with Rt eye pain and redness with watery discharge,no h.o trauma,itching,O/E there is diffuse
congestion in the conjunctiva and watery discharge what you'll do:
a. give Ab
b. give antihistamine
c. topical steroid
d. refer her to the ophthalmologist
???????????????
52)

mths baby with crying episodes+current jelly stool,looks slightly pale,signs of obstruction wht is your Mx:
a. barium enema
b. immediate surgery
c. I.v fluid & wait for resolution
The correct answer is a
Intussusceptions :
Condition in which one portion of bowel invaginates into an adjacent segment "usually proximal to ileocecal
valve" . most common in first two years of life " usually between 3 months and 3 years of life" . Abrupt onest ,
colicky abdominal pain , often accompanied by flexed knee and vomiting , ( one-off pain ) child may appear well
between episodes. Classic triad : abdominal pain , vomiting , blood per rectum" only one third of pt." . Late signs :

bloody mucus in stool "currant jelly stool" , abdominal tenderness , palpable "sausage- shape " . RUQ abdominal
mas.
Investigation & treatment :
-correct any volume
and electrolyte abnormality and check ( cbc)
Abdominal film may be normal in early stage , and
see obstruction , perforation in
late stage, US see " target sign"
In setting of high clinical suspicion >>> air-contrast barium enema, should be performed without delay. As
diagnostic in 95% of cases and curative in 80% of cases perform surgical reduction of gangrenous bowel.
53)

17. Pt with asymptomatic Trichomniasis:


a. treat her anyway regardless
b. treat her if she is symptomatic only
The correct answer is a
-treatment of trichomnias :
Single dose of metradinazole for symptomatic and asymptomatic pt. and
treatment partner ( because it is
sexual transmitted disease )
But not treatment of asymptomatic
trichomniasis in first trimester of pregnancy. (contraindication in pregnancy )

54)
pt with gonorrhea infection what else you want to check for
a. Clamydia trachomatis
55)

In battered women which is true:


a. mostly they come from poor socioeconomic area
b. usually they marry a second violent man
c. mostly they come to the E/R c/o..
d. mostly they think that the husband respond like this because they still have

strong feeling for them

Answer ( d )
Battered women is Women who are physically and mentally abused over an extended period, usually by a
husband or other dominant male figure. Characteristics of the battered woman syndrome are helplessness,
constant fear, and a perceived inability to escape. So I think choice d is correct answer

56)

Mother who is breast feeding and she want to take MMR vaccine what is your advice:
a. can be given safely during lactation
b. contain live bacteria that will be transmitted to the baby
c. stop breast feeding for 72 hrs after taking the vaccine
The correct answer is a
MMR : contraindication during pregnancy , and women should be avoid pregnant in 4 weeks followed MMR
vaccine . MMR is safe during lactation.

57)

Regarding peritonitis:
a.Complicated appendectomy the cause is anerobe organism
b. rigidity and the cause is paralytic ileus
c. can be caused by chemical erosions
d. ..
The correct answer is c

58)

the most effective thing regarding counseling:


a. family rapport
b. well adjusted appointment before counseling
c. .
?????

59)

Pt. has DM and renal impairment when he had diabetic nephropathy:there is curve for albumin
a. 5y
b. 10y
c. 20y
d. 25y
The correct is b

60)

Pt has alzahimar disease and halusination and delusion ttt:


a. Halopridole

Psychotic symptoms (e.g. hallucinations and delusions), agitation and aggressive behavior are common in
patients with Alzheimers Disease. A study suggests that haloperidol at a dose of 2-3 mgs/day is effective and
well tolerated by most patients.
61)

Pt has alzahimar disease and halusination and delusion ttt:


a. Halopridole
Psychotic symptoms (e.g. hallucinations and delusions), agitation and aggressive behavior are common in
patients with Alzheimers Disease. A study suggests that haloperidol at a dose of 2-3 mgs/day is effective and
well tolerated by most patients.

62)

Generalize anexity disorder best ttt:


a. SSRIs
b. tricyclic A D

The correct answer is a

63)

Major depression management:


a. Intial therapy even sever

Management of major depression disorder:


1-pharmacotherapy:
effective in 50 70% .allow for 2-6 weeks to take effect , treat more than 6 months ( SSRI, TCAs, MAOIs).
2-psycotherapy: psychotherapy combined with antidepressant is more effect than either treatment alone.
3- Electroconvulsion ( ECCT ).
4- phototherapy:
effective for pt. who has a seasonal pattern.

64)

65)

Psychiatric pt with un compliance of drugs ttt:


a. depro halopredol

Pt. with salpingitis and there is swelling in pelvis in posterior fornex and it is fluctuant m:
a. Colpotomy
b. Laproscopic
The correct answer is a (I'm not sure )
colpotomy, also known as a vaginotomy, is a procedure by which an incision is made in the vagina.
Purpose: A colpotomy is performed either to visualize pelvic structures or to perform surgery on the fallopian
tubes or ovaries.

66)

Child swallowing battery in the oesophegus, management?


a. broncoscoby

67)
In the appendisitis the histology is:
a. leukocyte in muscle
b. layer of lymphoid
c. tumor
d. plasma cell
the correct answer is a

in appendicitis : neutrophil exudation throughout mucus, submucus, and muscularis


68)

Salpingitis and PID on penicillin but not improve the most likely organism is :
a. Chlamydia trachomatis
b. Neisseria gonorrhoeae
The correct answer is b
Empiric antibiotic regimens should be aimed at treating likely causative agents, that is, N. gonorrhoeae, C.
trachomatis, genital mycoplasmas, and bacterial vaginosis-associated endogenous microflora. The latter include
anaerobic (Bacteroides and Prevotella species and anaerobic streptococci) as well as aerobic organisms (G.
vaginalis, E. coli, and facultative streptococci). Except for N. gonorrhoeae and some anaerobes, resistance is not
yet a clinical problem.

69)

Wound at end inflammatory phase when:


a. Epithelial tissue formation
b. Angiogenisis
c. when the wound clean
d. Scar formation
The correct answer is b

70)

Pt. come with history of tinia capitis ttt:


a. tar shampoo
b. Fluconazol

The correct answer is b


Newer antifungal medications, such as ketoconazole, itraconazole, terbinafine, and fluconazole, have been reported as
effective alternative therapeutic agents for tinea capitis. Of these agents, itraconazole and terbinafine are used most
commonly.
71)
Colon cancer with stage 3 give the chemotherapy:
a. As soon as possible
b. 1 month
The correct answer is a (I'm not sure because MCQs not complete )
Treatment for Stage 3 Colon Cancer
Treatment for stage 3 colon cancer generally consists of a surgical resection followed by chemotherapy. In a
surgical resection, a surgeon removes the part of the colon affected by the tumor and joins the remaining healthy
sections together to form one long, healthy piece.
The standard chemotherapy regimen used to be six months of treatment with 5-FU and leucovorin, but that
"cocktail" was developed decades ago and is seldom used anymore. there are many new chemotherapy regimens
available for stage 3 colon cancer.
72)
Sickle Cell Anemia give prophlaxis:
a. Penicillin
b. Iron
The correct answer is a
TREATMENT
Acute crisis: Analgesia and hydration.
Hydroxyurea to the amount of fetal hemoglobin.
H. influenzae and pneumococcal vaccines; prophylactic penicillin for
Children 5 years of age.
Acute chest syndrome: Respiratory support and exchange transfusion.

73)

diagnosis of thalasimia minor:


a. Hb A2 and Hb f
b. Microcytosis
the correct answer is a

74)

Pt. with MCV decrease and reticulocyt decrease iron deficiency anaemia investigation:
a. Ferritin level and TIBC and serum iron

75) Born BCG


a. 1month hepatitis b oral polio dpt
b. 2month s
c. 3month s
d. 9 to 12 month
mmr
understand th Q ( may the writer Q missed some information )

????? I can't

76)

E.histolytica cyst is destroyed by:


a. Freezing
b. Boiling
c. Iodine treatment
d. Chlorine
The correct answer is b

Amebiasis (or amoebiasis) is the name of the infection caused by E. histolytica.


To help prevent infection:
Avoid raw vegetables when in endemic areas as they may have been fertilized
using human feces.
Boil water or treat with iodine tablets
77)
Treatment of mania that does not cause hepatotoxicity
a. Lithium
carbamazepine
d. lamotrigine

b.
c. valporic acid

The correct anser is not a


Treatment of mood disorder :
Lithium : hepatotoxicity
carbamezapine : agrnulocytosis
asid : neural tube defect

valporic

The correct answer is a


78)

Patient present with high blood pressure (systolic 200) , tachycardia , mydriasis , sweating . what is the toxicity:
a. Anticholenergic
b. Sympathomimetic
c. Tricyclic antidepressant
d. Organophosphorous compounds
the correct answer b
Uses of Sympathomimetics: To treat and prevent reversible bronchospasm associated with bronchial
and nocturnal (nighttime) asthma, chronic bronchitis,emphysema, exercise-included bronchospasm and other
obstructive airway disease of the lungs.
To treat serious allegoric reactions (epinephrine injection only)

79)

the maximum body lenght will be reached after menarch by


a -6 months
b. 1 year

A child will have also reached her final adult height about two years after menarche.
80)

Old male with tender knee , pain , crepitus . the diagnosis:

a. Osteoarhritis
Ankylosin spondylitis

b.
c. Rheumatoid

the correct answer a


Osteoarhritis
OARTHRITIS (OA)
A chronic, noninflammatory arthritis of movable joints (e.g., DIP joints).
Characterized by deterioration of the articular cartilage and osteophyte
formation at joint surfaces.
Risk factors include a _ family history, obesity, and a history of joint
trauma.
Hx/PE: Crepitus; decreaserange of motion (ROM); pain that worsens with activity
and weight bearing but improves with rest.
81)
Mother has baby with cleft palate and asks you what is the chance of having a second baby with cleft palate or
cleft lip ,
a. 25%
c. %1

b. 50%
d. 4%

the correct answer is d


82)
1 liter fluid deficit equals :
a.1 kg
.
Liter of fluid deficit equal 1 kg of fluid as hydration protocol

83)
6 years child was born to HBS positive mother is HBS positive , he was only vaccinated by BCG after birth , what
you will give him now :
a. HBV + oral polio + DTP + hib
b. HBV + oral polio + dt + MMR +hib
c. HBV + oral
polio + Dt + MMR
?????????? all the above are wrong
84) Treatment of papules or pustules:

a.Topical benzoyl
b.Peroxide plus
topical antibiotics, mainly clindamycin or erythromycin.
c. In severe cases, intralesional
steroid injection or oral antibiotics, such as tetracycline or erythromycin may be added.
The correct answer b
TREATMENT
Mild acne: Topical clindamycin or erythromycin; benzoyl peroxide; topical
retinoids.
Moderate acne: The above regimen plus oral antibiotics such as tetracycline.
Severe nodulocystic acne: Oral isotretinoin (Accutane).
84)
Previously healthy female patient presented to ER with dysnea , anexiety , tremor , and she breath heavily , the
symptoms began 20 minutes before she came to ER , in the hospital she developed numbness periorbital and in her
fingers , what you will do
a. Ask her to breath into a bag
b. Take blood sample to look for alcohol toxicity

the correct answer is a


85) What is the most important in councling
a. Exclude physical illness
b. Establishing rabbot
c. Family
d. Schedule appointement
?????

86)

In breaking bad news

a. Find out how much the patient know


Find out how much the patient wants to know
the correct answer a
Robert Buckman's Six Step Protocol for Breaking Bad News
1. Getting started.
2. Finding out how much the patient knows.
3. Finding out how much the patient wants to know.
4. Sharing the information.
5. Responding to the patients feelings.
6. Planning and follow-through.
87) 35 year female with bilateral breast pain , that decrease after menstruation , the breast is nodular
with prominent 3 cm mass subareolar , axillary lymph nodes are not enlarged , what you will do
a. Mammography followed by US
b. See her next cycle
c. Fine needle biopsy followed by tissue studies
the correct answer is a or c

b.

88)

Patient with dysuria , frequency , urgency , but no flank pain , what is the treatment

a. Ciprofloxacin po od for 3-5 days


b. Norfocin po od for 7 14 days

the correct answer is b


89) Male with itching in groin erythematous lesions and some have clear centers , what is diagnosis :
a. Psoriasis
b.
Tinea curis
c.
Erythrasma
the correct answer is b

Patients with tinea cruris report pruritus and rash in the groin. A history of previous episodes of a similar
problem usually is elicited. Additional historical information in patients with tinea cruris may include recently
visiting a tropical climate, wearing tight-fitting clothes (including bathing suits) for extended periods, sharing
clothing with others, participating in sports, or coexisting diabetes mellitus or obesity. Prison inmates, members of
the armed forces, members of athletic teams, and people who wear tight clothing may be subject to independent
or additional risk for dermatophytosis.Large patches of erythema with central clearing are centered on the inguinal
creases and extend distally down the medial aspects of the thighs and proximally to the lower abdomen and pubic
area.90) Vasoconstrictive nasal drops complication
a.Rebound phenomenon
Topical decongestant use should be limited to two to three days because rebound rhinitis can occur after 72
hours of use. The use of topical decongestants may occasionally be complicated by nosebleeds, agitation,
insomnia and worsened hypertensive control in patients with preexisting hypertension.
91)

One of the following decrease chance of colon cancer : a. Zinc b. Vit. E c. Vit C d. Folic acid

The answer is d
92) Which drug is contraindication in Acute cholecystitis :
Naproxen
Acetaminophen
Morphine
--------morphin

93) Vertigo & .. is caused by which of the following drug ?


a. Aphotercine B
b. Pnicilline reaction
c. INH
drugs that cuses vertigo are 1-gentamicin 2-diuratics 3co-trimoxazole 4metronidazole.so ithink anser
missed
94) - 31-Pregnancy test +ve after :
a- one day post coital
b- 10 day after loss menstrual cycle ??
c- One wk after loss menstrual cycle
they said c
94)
A question about an old diabetic who experienced a sudden painless unilateral loss of vision, eye
examination revield retinal hemorrhages, vascular changes and macular oedma...what is the cause of
his blindness 1-retinal artery occlusion 2-retinal vein occlusion 3-diabetic retinopathy
The answer is 2
me
95)- 50-55 year old lady with rosacea not responding to treatment , for her look you should advice: 1Using systemic vasodilators 2- Topical steroids 3- Green base Make up 4-Sun exposture
the answer is 3 they said
95) .HIV pt with purple difuse lion in the skin and oral cavity treat?a.antibiotic b.topical steriod c.chemo
&radio theraby7
96) which one of the following is true about exercise :
a- exercise decrease HDL
b- exercise increase C reactive protein
c- not useful in central obesity
d- to get benifetyou have to exercise daily
they said b
97) -In which group you will do lower endoscopy for patients with iron deficiency anemia in with no
benign cause:
-male all age group
-children
-permanupausal women
-women + OCP

They said: male all age group


98) -Pregnant women present with a mass in her mouth bleeding when brush her teeth
by examination mass 3x2 cm, diagnosis :
- aphthous ulcer.
- .....cancer
- .....granuloma
They said .....granuloma
98) - case scenario , baby present with unilateral deformity in the foot appear when it is become the
weight bearing is in the other foot but when it is the weight bearing the deformity disappear ,the
patient has defect in dorsiflexion of that foot .......I think they are taking about ( club foot )
treatment :
A-orthopedic correction ...?
B-shoe....
C-surgery ....
They said :a
99) Patient is presented with hand cellulitis and red streaks in the hand and tender axillary
lymphadenopathy.This conditionis more likely to be associated with:
Malignancy
Pyoderma
Neuropathy
Lymphangitis
I said Lymphangitis
100) female patient onthe3rdweek postpartum. She says to the physician that the frequently visualizes
snakes crawling to her babys bed. She knows that it is impossible but she cannot remove the idea from
her head. She says she wakes
up around 50 times at night to check her baby. This problem prevents her from getting good sleep and
it started to affect her marriage. What is this problem
she is experiencing?
An obsession
Ahallucination

Apostpartumpsychosis
Delusion
I said An obsession
101) In holding breath holding which of the following True:
a. Mostly occurs between age of 5 and 10 b. Increase Risk of epilepsy c. A known precipitant cause of
generalized convulsion d. Diazepam may decrease the attack
They said c
102) the following more commone with type2 DM than type1 DM:
Weight loss
Gradual onset
Hereditary factors
HLA DR3+-DR4
They said Hereditary factors
103) pt kown case of migraine what is complication exepected for him in future a\schizophrenia b\
depressioin c\suscide attempt ,td ,hp] jhkd lhlj.;vhi,
I said b
104) mammogram can detect deep breast cancer before breast self examination by how many years???
one,2,3,4
I said 2years
105) Patient came with HTN, KUB shows small left kidney, arteriography shows
renal artery stenosis, what is the next investigation:
Renal biopsy
Renal CT scan
Renal barium
Retrograde pyelography
106) Vertigo & .. is caused by which of the following drug ?
a. Aphotercine B
b. Pnicilline reaction
c. INH
drugs that cuses vertigo are 1-gentamicin 2-diuratics 3co-trimoxazole 4metronidazole.so ithink anser
missed
107) 40 yrs for pap smear you tell her
a\should be annualy
b\every 5 yrs

c\no need
d\after 3 normal resultes no need at all
107) WHAT IS the most likely to develop DVT?
a\opin knee surgery
b\prevously PE in pregnancy
c\past DVT
They said c
108) QUESTION ABOUT best excercise for OA
I said >>>walking, swimming, and cycling
109) Allopurinol
a- use in acute phase
b- b-it is uricosuric
c- c-contraindication in chronic renal disease
d- decrese uric acid renal stone
the answer is d
110) old pt ,she have MI and complicated with ventricula tachycardia
then from that time reciveBuspirone he came with fatige>>>>normotinsive , pulse was 65 what INX
must to be done
a- thyroid function
b- liver and thyroid
c- liver & kidney
the answer: I said c
111) bad breath smell with seek like structure, no dental caries & Ix are normal, what's the likely cause:
- cryptic tonsillitis
- Sojreen's synd.
The answer is a >>> Bad Breath Tonsil stones are particles of bacteria and dead cells lodged within the
"crypts" of the tonsils. They tend to cause extreme bad breath, which is obvious to the sufferer and
others. Unfortunately, the odor cannot be easily eliminated and does not respond well to over-thecounter mouthwashes or halitosis treatments.
112)
QUESTION ABOUT best excercise for OA ?
My answer walking, swimming, and cycling
113) old aged female with atypical squamous cells of undetermined significance (ASCUS) on pap
smear, started 30 day ttt with estrogen & told her 2come back after 1 weak, & still +ve again on pap
smear, what's next:
- vaginal biopsy
- endometrial biopsy

- syphilis serology
In 4th is a
113)
24 y. Female with new Dx of DM2, she weared glasses for 10 years, you will advice her to follow
ophthalmic clinic every:
- 6 months
- 12 months
- 5 years
- 10 years
In 4th is b
114)
chickpeas.kidney beans and lentils contain which element of following
bromide
chromium
iron
selenium
In 4th is c
115)
a case of clamydial eye infection
Treatment consists of systemic antibiotics; topical antibiotics are relatively ineffective in the treatment of
this eye disease.
- Recommended treatment, which is given for 3-6 weeks, includes oral tetracycline (500 mg qid), oral
doxycycline (100 mg bid), or oral erythromycin stearate (500 mg qid).
- Azithromycin can be given as a single dose of 1 g, which can be increased to 2 g if Neisseria
gonorrhoeae is suspected.
- Tetracyclines are avoided in children younger than 7 years and in women who are pregnant or
breastfeeding.
116) Lucman test : in ortho
The Lachman test is an orthopedic test used for examining the anterior cruciate ligament (ACL) in the
knee
116)
about shoulder that is Adducted and internally rotated (what is the mechanism of dislocation)
Posterior dislocations are occasionally due to electric shock or seizure and may be caused by strength
imbalance of the rotator cuff muscles. Patients typically present holding their arm internally rotated and
adducted, and exhibiting flattening of the anterior shoulder and a prominent coracoid process
117)
Tineacapitis RX.
1-start Nystatin
2-wood's lamp (true)
My answer is grisoflavin
118)
Rosacea case (redness patch on face with talangectasia ) what is the ttt :

Dyoxycyclin ??????????
119)
Q12/ child smile at
at birth
1month
2 months
6 months
Mine is c
120)
infertile women for 3 years with dysparunia
endometritis
Salpengitis
3-endometriosis (True) in 4th
121)
OCP protective Against :
Breast ca
Ovarian Ca
Endometrial
I said b
122)
Typical scnario about migraine and pt doesn't want DAILY medication :
1- bio feedback
2- BB
3- CCB
4-inhaled ergometrin
They said 1
123)
lactating mom recently dx to have epilepsy on Phenobarbital , her child is
10 months now , whats appropriate to tell her
1- stop immediately
2- wean him for 2 weeks
3- give after 8 hrs of
4- BF as much the baby and mother want
They said 1
124)
pregnant pt want to take varicella vaccine, what you will tell her ?
That is a live vaccine
It is ok to take it
Mine a

125)
pt with tingling of the little finger, atrophy of the hypothener, limitation of the neck movement, Xray shows degenerative cervicitis, EMG study shows ulnar nerve compression, what will you do: Surgical
cubital decompression(my ans) Cervical CT scan Nsad
Phisotherapy
most commen of bleeding on postmenoposal women
A-carvical plup
B- utrine atony tru 100 %
Mine However, the most common cause of bleeding in these women is atrophy of the vaginal mucosa or
endometrium [3]. In the early menopausal years, endometrial hyperplasia, polyps, and submucosal
fibroids are also common etiologies
126)
pregnant never did check up before , her baby born with
hepatosplenomegaly and jaundice :
a-Rubella
b-CMV
c-HSV
d-Toxoplasmosis
they said b
127)
What the best method for prevention dieses :
Immunization
Teaching individual how to protect them self
they said b
128)
Appendcites proflaxx
Metronidazole
Ceftroxone( my answer)
Cefroxon
I said Cefazolin provides adequate coverage for most other types of procedures.
129)
anal fissure most commen site
Posterior( my ans)
Anterior
Anal fissures usually extend from the anal opening and are usually located posteriorly in the midline,
probably because of the relatively unsupported nature and poor perfusion of the anal wall in that location.
130)
A 5 year old child came with earache on examination there is fluid in middle ear and adenoid
hypertrophy. Beside adenoidectomy on management, which also you should do:
Myringotomy

Grommet tube insertion( MY answer)


Mastidectomy
Tonsillectomy
131)
there is outbreak of difteria and tetanus in community , regaring to pregnant woman:
contraindication to give DT vaccine
if exposed , terminate pregnancy immediately
if exposed , terminate after 72 hour
give DT vaccine anyway
I said d
132)
Most common cause of intra cerebral hemorrhage
Av malformation( my ans)
Pre exicting anurezem
The most common cause of intracerebral hemorrhage is high blood pressure (hypertension). Since high
blood pressure by itself often causes no symptoms, many people with intracranial hemorrhage are not
aware that they have high blood pressure, or that it needs to be treated. Less common causes of
intracerebral hemorrhage include trauma, infections, tumors, blood clotting deficiencies, and
abnormalities in blood vessels (such as arteriovenous malformations)
133)
Lactating mother with mastitis trratment:

Doxcycyclin
Ceftroxone
Cefoxine
Metronidazle
They said b
134)
patinet knwon case of cancer , came several times to clinc with pain in different sites . wat is the
BEST thing to do:
- give analgesics
- send home nothing to do
- refer the patinet to psychiatrist
they said refer
135)
after colectomy for ca colon follow up every
-3month
-6month

-9month
-12 month
I said a
136)
patient was treated for glaucoma now presented with SOB , ... The drug reasons able for these
symptoms :
Timlol
Pilocarpine
I said b
137)
young lady with emphysema
A1 anti-trypsin def
In 4th
138)
lab values all r normal except Na ( hyponitremia ) treatment
NS with kcl at 20 cc / hour
NS with kcl at 80 cc\ hour
1/2 ns ...
In 4th b
139)
45 - 27 years old with DM 2 she already wears glasses u will follow up her after :
6 months
12 months
In 4th b
140)
All cause ear pain except:
a- Acute otitis media
b- b- Dental caries
c- c- Vestibular neuritis
d- Temromandibular joint arthritis
I said c
141)
Pt presented with severe hypothyroidism & serum sodium = 108. What do u do?
Intubate, give 3% sodoium then treat hypothyroidism status
treat hypothyroidim& monitor S.NA level every 6 hours
Give 3% sodium, hydrocortisone & treat hypothyroidism status
it is c in 4th
142)
Which breast disease is Bilateral:
Lobular carcinoma in 4th
143)
Regarding pterygium:
of systemic cause
b- causes blindness
c- due to avitemenosis A

e- needs surgical intervention


it is b in 4th
144)
A 10 YO was diagnosed with rheumatic fever without any defect to the heart. You will tell his parents
that he needs to take prophylactic antibiotics for how many years?
a- 5 months
b- 3 years
c- 6 years
d- 15 years
TABLE 3.
Duration of Secondary Prophylaxis for Rheumatic Fever
Evidence
rating*

Type

Duration after last attack

Rheumatic fever with carditis and residual heart


disease (persistent valvular disease)

10 years or until age 40 years (whichever is longer);


lifetime prophylaxis may be needed

1C

Rheumatic fever with carditis but no residual heart


disease (no valvular disease)

10 years or until age 21 years (whichever is longer)

1C

Rheumatic fever without carditis

5 years or until age 21 years (whichever is longer)

1C

*American Heart Association evidence ratings: 1C = case studies, standard of care, or consensus opinion that a procedure
or treatment is beneficial, useful, and effective.
Clinical or echocardiographic evidence.
Adapted from Gerber MA, Baltimore RS, Eaton CB, et al. Prevention of rheumatic fever and diagnosis and treatment of acute
Streptococcal pharyngitis: a scientific statement from the American Heart Association Rheumatic Fever, Endocarditis, and
Kawasaki Disease Committee of the Council on Cardiovascular Disease in the Young, the Interdisciplinary Council on
Functional Genomics and Translational Biology, and the Interdisciplinary Council on Quality of Care and Outcomes Research:
endorsed by the American Academy of Pediatrics. Circulation. 2009;119(11):1547.

TABLE 4.
Secondary Prevention of Rheumatic Fever

Agent

Dosage

Penicillin G benzathine

Patients weighing 27 kg (60 lb) or less: 600,000


units IM every 4 weeks

Evidence
rating*
1A

Patients weighing more than 27 kg: 1,200,000


units IM every 4 weeks
Penicillin V potassium

250 mg orally twice daily

1B

Sulfadiazine

Patients weighing 27 kg or less: 0.5 g orally once


daily

1B

Patients weighing more than 27 kg: 1 g orally once


daily
Macrolide or azalide antibiotic (for patients allergic to
penicillin and sulfadiazine)

Varies

1C

IM = intramuscularly.
*American Heart Association evidence ratings: 1A = evidence from multiple randomized trials or meta-analyses that a
procedure or treatment is beneficial, useful, and effective; 1B = evidence from a single randomized trial or nonrandomized
studies that a procedure or treatment is beneficial, useful, and effective; 1C = case studies, standard of care, or consensus
opinion that a procedure or treatment is beneficial, useful, and effective.
Administration every 3 weeks is recommended in certain high-risk situations.

Macrolide antibiotics should not be used in persons taking other medications that inhibit cytochrome P450 3A, such as
azole antifungal agents, human immunodeficiency virus protease inhibitors, and some selective serotonin reuptake
inhibitors.
Adapted from Gerber MA, Baltimore RS, Eaton CB, et al. Prevention of rheumatic fever and diagnosis and treatment of acute
Streptococcal pharyngitis: a scientific statement from the American Heart Association Rheumatic Fever, Endocarditis, and
Kawasaki Disease Committee of the Council on Cardiovascular Disease in the Young, the Interdisciplinary Council on
Functional Genomics and Translational Biology, and the Interdisciplinary Council on Quality of Care and Outcomes Research:
endorsed by the American Academy of Pediatrics. Circulation. 2009;119(11):1547.
145) TRUE about congenital hip dislocation :
-abducting an infants flexed hip causes palpable click ortalani test
Is a as in 4th
146)patient known to have CHF , came with fatigue & muscle weakness , Na :123 ,
K: 3 , how ull manage ?
-normal saline with rate 80 /hour +40 mickcl
-normal saline with rate 20/hour +5mic kcl
-1/2normal saline with rate 80 /hour +40 mickcl
-1/2normal saline with rate 20/hour +5mic kcl
It is c in 4th .
147) most common cause of conductive hearing loss :
-acute otistis media
-Meniere disease
- perforated tympanic membrane
It is a in 4th
148) SE of sulfodizene :
Leucopenia
as in 4th
149-qs about vesicocele
U should know if its repair of upper ant of the vaginal wall so the defect is
vesicocele, if its lower anterior urethrocele , if upper postenterocele , if
lower post rectocele

150) pt asking u why instead of doing self breast exam. Every month not to do
mamogaraphyyearly , what ull say :
-mamography only detect deep tumor
-mamography and self exam. Are complementary
-self breast exam are better bcz it detect early tumor
- mammography are only for palpable masses
It is b in 4th
151) drug are contraindicated in ttt of glaucoma :
Timolol
Pilocarpine
NSAID
2 new drugs I was confused , I dont remember their names
It is c in 4th
152) famouse actor diagnosed with rosacea with tenlagectasia , he concerns about his look , what is your advice regarding
his treatment:
Tropical steroids
Laser surgey
retinoids
vasodilator to reduce tenlagectasia
I said b
153) baby at 6 months , what he can do ?
-sitting without support
-role from supine to prone position
- role from prone position to supine
It is c in 4th
154) What feature of schizophrenia suggest good prognosis ?
-family hx of scz
-no precipitating factors
-presence of affecting symptoms
-early onset
It is c in 4th
155) drug used in smoking scessation c/I in pt :
-hx of seizure
as in 4th
156) the most effective way in health education :
Mass media
Group discussion

Individual approach
It is c in 4th
157) child with white yellow mouth lip n erythrematous base with gingivitis :
HSV
EBV
CMV
B in 4th
158) What is contraindication for giving welbutrin in smoking sessation ?
History of seizure
Hemolytic anemia
A in 4th
159) Acyanotic middle age man radiologycally come with prominent pulmonary
arteries and vascular marking ,most likely Dx?
VSD
ASD
Coarticarion of the aorta
Truncus arteriosis
Pulmonary valvular stenosis
B in 4th
160) Patient come you find perforated tympanic membrane with foul withish
discharge dX?
Otoseclerosis
Otitis externa
Cholestitoma
C in 4th
161) Patient 42 years with 5 days history of skin eruption involving the hand and
soles (no other information)dx?
Erythema mutiforme
Fixed drug eruption
Pytriasis rosea

A in 4th
162) Eye screening in DMI
Now and annually
Now and every 10 years
After 5 years and annually
C in 4th
163) Sodium content in normal saline (0.9)
50
70
90
155
200
D in 4th
164) patient complain of irregular periodand excessive fasial hair .her mother had
the same. BMI 36 normal estrogen increase testerone increase LH and
decreased FSH,and her urine shows 17 hydroxysteroid Dx
Chushing
Polycystic ovary
Adrenal adenoma
C in 4th
165)Patient work outside in hot weather 42C came to ER with muscle pain and
cramps of the lower limb ,on examination he is alert ,cooperative ,temp 38
,Managment
Oral electrolyte replacement
Internal cold water
Warm intravenous fluid
tepid water
a in 4th
166) patien work in hot weather come with clammy cold skin ,hypotensive
Tachycardiac ?
heat stroke
heat exhusion
a in 4th
167) patient came with osteoprotic thorasic vertebral fracture t score for vertebra
-2.6 z score
the hip -1.6 and z score 0.9 according to who classification the patient has
osteopenia

osteoporosis
established osteoporosis(my answer)
normal bone mass
WHO classification :
(Normal boneT-score greater than -1
OsteopeniaT-score between -1 and -2.5O
steoporosisT-score less than -2.5
Severe (established) osteoporosisT-score less than -2.5 and 1+ osteoporotic
fractures)
167) what is questioneer used to diffrentiet between sleep apnea and snoring?
Mitchigan
Epworth (the correct )
Cooner
B in 4th
168) Female with Hx of PID and treated with ABs she came later with fever and pain
on examination there was a mass, fluectuent (they mean abcess ) in a cul-de
sac !!
what is ur next step?
colpotomy
laparotomy
laparoscopy
Pelvic US
A in 4th
what's true about antipsychotics ?
a) predominantelymetabloized in the liver
b) Carbamazepin as a single dose os better than divided doses
A in 4th
169) Al hydroxide+ Mg hydroxide inhibits the intestinal absorption of which drug?
a) Tetracycline
sorry I don't remember the rest
a in 4th
170) in epidemiological investigation best thing to do 1st:
a) good sample
b) count those who have the disease
C)verifying diagnosis
C in 4th
171) if the liklihood ratio is 0.3 what does that mean??

172) spiral fraction in child > open reduction and interna fixation
173) fishy viginal discharge > bacteria vaginosis
174) In the appendisits the histology is:
a. leukocyte in muscle
b. layer of lymphoid
c. tumor
d. plasma cell
the correct answer is a in 4th
in appendicitis : neutrophil exudation throughout
mucus, submucus, and muscularis
175) exercise recommended for patients with CAD. is
a. isometric
b. isotonic
c. yoga
176) lesion in brain for taste
a. Temporal lobe
b. Cerebellum
c. Parietal lobe
d. Occipital lobe
177) brain cell death in alzehimer disease ( not recognized his wife and
fighting whit her )
a. Temporal lobe
b. Cerebellum
c. Parietal lobe
d. Occipital lobe
178) Benign tumors of stomach represent almost :
A. 7 %
B. 21 %
C. 50 %
D. 90 %
A in 4th
179) Child with atopic dermatitis at night has stridor plus barking
cough on and off from time to time, diagnosis is:
a) BA
b) Croup
c) Spasmadic Croup
d) ?

The correct answer is c in 4th


Spasmadic croup : recurrent sudden upper airway obstruction which
present as sridor and cough . approximately 50% of children have atopic
disease.
180) As doctor if you see patient and you face difficulty to get
accurate information from him the best tactic to do it is:
a) Ask direct question
b) Ask open question
c) Control way of discussion
d) ?
The correct answer is a in 4th
181) As doctor if you see patient and you face difficulty to get
accurate information from him the best tactic to do it is:
a) Ask direct question
b) Ask open question
c) Control way of discussion
d) ?
The correct answer is a
http://www.aafp.org/afp/2005/1115/p2063.html
182) 60 y/o male Dx to have acute pancreatitis ( there was values for
ranson criteria that I couldnt memorize ) what is the appropriate
nutrition :
a- TPN
b- Regular diet with low sugar
c- High protein ,high ca , low sugar
d-Naso-jujenal tube
d in 4th
183) ectopic pregnancy in fallopian tube what you well do
wait and observe
laborotomy
albroscopy
A they said
184) which one of the following anti TB medication is consider as drug induce SLE
A/ ethambutol
b/INH
c/streptomycin
d/rifampin

B IN 4TH
185) endemic means
a/ spread of disesas in incidence all the time
b/it cause by virulent pathologic organism
c/ spread of disesas from country to country by carrier
d/rapid spread of disesase
e/ there is very low incidence
A in 4th
185) before 14 d the child was bite ,nowdevelope lip swelling eryrhema ... ,
what type of hypersensitivity ?
a/type 1
b/type 2
c/thype 3
d/type 4
d in 4th
186) healthy femal came to your office complain of lesion in her vagina that
stared sice just 24 h . O/E there is cystic mass lesion non tener measure 3 cm on
her labia , what is the the most likely Dx :
...
A/ bartholincysy
B/Vaginal adenosis
C/ schic cyst
D / hygroma
C in 4th
187) adult healthy male came with tender red swelling on right hand up to
forarm and you found black head and large pore skin , he said it happen after
trauma to his hand 1 week back , the management shuld be :
A/ topcical antibiotic
B/ topical Antfungal
C/cryosurger
D/ Oral antibiotic the answer
acyually I dagnose this case as Hidraadenitissuppurativa but I am not suure
and as i know the ttt= mild = cold compressor ( was not in MCQ)
moderater =AB
sever= surgery
188) male, presented with pain in the posterior aspect of the thigh, he was
running long distance felt a pop in his thigh, on exam, tenderness, erythema,

and swelling, no defect what is the best treatment:


a- Surgery
b- Ice, rest, bandages, and elevation of the limb
c- Bandages only.
d-Splint.
B in 4th
189) Which congenital heart condition is the most common associated with
endocarditis
A/ VSD
B/ ASD
C/PDA
D/TOF
D in 4th
190) patient diagnose withDm type 2 and he is in your office to discusse with
him the plane to reduce his weight , you will told him to :
A/decrease calori intake in day time
B/decrease calori and increase fat
C/decrease by 500 kcal/kg per week( my answer correct )
D/ decrease 800 per day
C in 4th
191) Old patient with asthma and urine retention due to prostatic enlargement,
hypertensive (BP: 180/100) what's the most appropriate drug to control
hypertension?
A/ Labetalol
B/Phenalamine
in 4th
C/Propanolol
Same Q before but her with BA and different drug
192) preganant lady 38 wks GA with placenta previa
marginal with mild bleeding , the cevix is dilated cervix 2 cm
How to manage ;
A/ CS
B/spontiusdelvery
C/forceps delivery( most likely ) in 4th
D/ do amniotomy
193) whaich one of the following is true regarding the weight gain in
pregnancy:
A/Pregnant woman should consume an average calorie 300-500 per day( my answer most likely ) in 4th

B/ Regardless her BMI or body weight she should gain from 1.5 3 lb which represent the
baby's growth
194) Lichen planus most commonly found in :
A/Scalp
B/ Knee
C/ Buttocks
D/Mouth( my answer and most likely )
Mine d
195) Likelihood ratio of a disease incidence is 0.3 mean:
A/.large increase
B/small increase
C/no change
D/ small decrease
E/large decrease
D in 4th
196) All can cause gastric ulcer except:
A/ Tricyclic antidepressant.
B/ Delay gastric emptying.
C/ Sepsis.
D/ Salicylates.
E/Gastric outlet incompetent
They said a
197) 8 months child with 3 days fever 40 , vomiting , convulsion , poor feeding &
sleep , OE dehydrated , depressed ant frontanell, red ears ,no neck stiffness , his
3 year old sibling asymptomatic , whichof the following will give the
defenitiveDx :
a- CXR
b- CBC with deferential
c- blood culture ( my answer not sure , I think about sepsis ) in 4th
d- CSF analysis
e-suprapubic urine analysis
198) in PHC, from 50 child 10 got the disease on the 1st week, another 30 on the
subsequent 2 weeks, what is the incidence of the disease in that PHC?
a- 20%
b- 40%
c- 60%
d- 80% ( my answer ) , 10+30 /50 in 4th

e-90%
199) the name of Questionnaire that differenctite between the primary and
secondryapnea :
Straing name BUT the correct answer is horchover
as in 4th
200) which one is true Regarding appendicitis in the elderle:
A/ If the patient is afebrile this rules out appendicitis
B/WBC is often normal( my answer not sure but was most likely for me )
C/Rupture is not common
D/anemia is coming finding
As in 4th
201) What the best method for prevention dieses :
Immunization
Teaching individual how to protect them self
They said b
202) young lady has inlfamtory acne, she used mant treatments without improvment, finally she used accutane, what is the
most side effect you will tell her about:
- Teratogenicity
- Acne will getting worse before it progress
-Back pain
-Nause and vomiting
A they said
203) Baby c/o 5 times diarrhea , 7 times vomiting in last 24 hrs, dehydated , low BP , low PR and weak, with fever. on
investigation nomal , except high urine gravity,cause is :
Rota virus
E coli
Shigella
A they said
204) after colectomy for ca colon follow up every
-3month
-6month
-9month
-12 month
A I said
Pooled analysis form several large adjuvant trials reported that 85% of colon cancer recurrences occur within 3 years from
after resection of primary tumor. Therefore, patients with resected colon cancer (stage II and III) should undergo regular
surveillance for at least 5 years following resection. An update of American Society of Clinical Oncology (2005) recommends
physical examinations every 3-6 months for the first 3 years, every 6 months during years 4 and 5, and subsequently at the

discretion of physician and based on individual risk assessment.


Serum CEA level should be checked every 3 months in patients with stage II or III disease for at least 3 years and every 6
months in years 4 and 5. Computerized tomography (CT) of the chest and abdomen should be performed annually for at
least 3 years after resection of primary tumor. All patients with colon cancer should have preoperative or postoperative
colonoscopy to document absence of additional primary colon tumors or polyps. In the absence of high-risk pathology on
the first colonoscopy or increased susceptibility for colon cancer, follow-up colonoscopy should be performed at 3 years
after surgery and then, if normal, once every 5 years thereafter.
205) pt has Lt lower Abdominal pain , Fever , constipation CT reveals thickened
loop and little perianal fat , whats appropriate to do:
1- start AB
2- call the surgeon for immediate OP
3-give laxative
4- barium enema
B they said
206) lesion in brain for taste
a. Temporal lobe
b. Cerebellum
c. Parietal lobe
d. Occipital lobe
C they said
207) Eye screening in DMI?
Now and annually
Now and every 10 years
After 5 years and annually
The answer : type 1 after 5 years of diagnosis and annualy --type 2 after dignosis and annualy
208) pt going to surgery . afraid of DVT What we give ( cost effective .
and .and)
LMWH
UNFR. HEPARIN
ASPRIN
WARFARIN
B they said
209) brain cell death in alzehimer disease ( not recognized his wife and
fighting whit her )
a. Temporal lobe
b. Cerebellum

c. Parietal lobe
d. Occipital lobe
A they said >>> me also
210) femal child present with regreation of mild stone and social activity and 000000000000000000000 RETT syndrome
they said
211) All in hypokalemia exept:
Hyper osmolar coma
Phention toxicity
Musle paralysis
A they said
212) A middle age man presented with severe headache after heavy lifting
objects. His BP was high. He was fully conscious. Examination was otherwise
normal. the most likely diagnosis is:
a) Subarachnoid hemorrhage
b) Central HTN
c) Tension headache
d) Migraine
e) Intracerebral hemorrhage
d they said
213) 4 y/o child with diarrhea for 2 days is complaining of anal discomfort. Your
advice to the mother is:
A/Wash with soap and water after each episode of diarrhea.
B/Wash with cotton in warm water.- ( my answer ) in 4th
C/Put a clean napkin in the underwear.D/Change the underwear to a highly absorbent diaper
214) 4 Y/O Baby with scenario of ADHD, what is the best treatment in addition
to behavioral therapy:
a- Atomoxetine( my answer correct ) in 4th
b- Imiramine
215) The most cause of tinnitus:
A/ vitiligo
b- Sensory neural deafness
c- acute otits media
e-noising induce tinnitus. ( my answer ) in 4th
216) which one of the following anti TB medication consider drug induce SLE
The answer is INH in 4th
217) which on of the following OCP cause hyperkalemia :

The answer is ethynyl estradiol


218) Child presented with gum and nose bleeding and bruising all over the body
after an episode of URTI. Dx:
5) Henoch Scholein Purpura
6) Idiopathic thrombocytopenic purpura(sure) in 4th
7) Vitamin K deficiency
8) Hemophilia
219) the most common side effect of antipsycotic
a.alopecia
b. wt gain( correct ,, got 5/5) in 4th
c.hypotention
d.constipation
220) 26 y/o pt. k/c of depression taking (citalopram)for depression ,, presented
with ingestion of unknown drug ,,, on investigation she was found to have
metabolic acidosis and anion gab 18 ,,, what is the most likely drug she
ingested??
a. paracetamol
b. asprin
c. citalopram
d. amitriptyline
b in 4th
212) 19y/o not known to have any medical illness ,presented with fever, arthritis,
and rash mainly in the palms and soles ,,, he gave hx of illegal relationship ,,,
mostly he is having??
a.chancroid
b. 2ndry syphilis
c. chalmedia trachomatis
b I said
213) 60 y/o pt. presented with decrease vision bilt , specially to bright light on
exam he was having cupping with wedge shaped opacities ... he is having??
a.lentis sublexation
b.catract (my answer and i got 3/3) in 4th
c.open angle glaucoma
214) Child presented with gum and nose bleeding and bruising all over the body
after an episode of URTI. Dx:
5) Henoch Scholein Purpura
6) Idiopathic thrombocytopenic purpura

7) Vitamin K deficiency
8) Hemophilia
I said 6
215) A middle age man presented with severe headache after heavy lifting
objects. His BP was high. He was fully conscious. Examination was otherwise
normal. the most likely diagnosis is:
a) Subarachnoid hemorrhage
b) Central HTN
c) Tension headache
d) Migraine
e) Intracerebral hemorrhage
e they said
216) 38 week pregnant lady came to ER in labor, cervix 4.5 cm dilated, marginal
placenta previa. Management:
1) Wait and evaluate fetus
2) SVD
3) C/S
4) Forceps
5) Rupture membrane
3 I said
217) mother gave bitrh of baby with cleft lip and palate, she want to
get pergnant again what is the percentage of recurrence
a. 1%
b. 4%
c. 15%
b they said ---1%in cleft palat alone
218) Computer programmer, a case of carpet tunnel syndrome, how to splint:
f. Dorsiflexion( sure) in 4th
219) Gastrictomy post-op 1 day. He have temperature 38.8 & pulse 112. What is
the most common cause ?
a. wound infection.
b. inflammatory mediator in the circulation.
c. UTI
d. normal
b in 4th

220) man walking in street and saying bad words to stranger , he is not aware of
his
conditiond , what is the description :
flight of idea
Deprivation(got 5/5) in 4th
insertion of idea
loosening of association
221) Holding breath spell or holding ..which of the following is true
A)mostly occurs between age 5-10
B)increase risk of epilepsy
C)a known precipitant cuz of generalized convulsion my answer in 4th
D)diazepam may decrease the attack
e) can occur in absence of emotional upset
222) Most common organism causing cellulitis in the age 6-24 month
E. Strepto coccus
F. Heamophilus influ
G. Staph
223) pregnant lady with hyperthyroidism what you will give her :
propylthiuouracil
methamazole
B blocker
Radioactive iodine
a in 4th
224) The most common cause of tinnitus:
A/ vitiligo
b- Sensory neural deafness
c- acute otits media
d- noising induce tinnitus.
D I said
225) Football player injured in the lateral side of his LT knee, presented to
you with sever knee pain, PE there is swelling in the medial aspect of the
ltknee ,valgus test showed free mobility but lachman test and McMurray's
test are negative . what's your diagnosis:
f- Lateral collateral ligament injury
g- Medial collateral ligament injury
h- Patellar fracture
i- Medial menisci injury

j- Lateral menisci injury


g in 4th
226) Patient with rheumatoid arthritis came to came to you and asking
about the most effective way to decrease joint disability in the future, your
advice will be:
c- Cold application over joint will reduce the morning stiffnesssymptoms
e-Disease modifying antirheumatic drugsare sufficient alone
c in 4th
227)Most common cause of recurrent tonsillitis :
f- Group B streptococcus
g- EBV
h- Bacteriod
i- Rhino virus
j- Parainflunza virus.
G in 4th
228) Patient with disc prolaps will have:
e- Loss of ankle jerk
f- Fasciculation of posterior calf muscles.
g- Loss of Dorsiflexion compartment of the foot.
h- Loss of the sensation of the groin and anterior aspect of the thigh
g in 4th
229) 6 month child , difficulty in breast feeding , active pericardium,
pansystolicmurmer s1 , loud s2
ASD
large VSD
MR
AR
PDA
B in 4th
230) Child with ear congested , opacity , recureent URTI , o/e NEED
adenectomy , beside adenectomy u must do:
tosilectomy
maryingotomy
government tube
b in 4th
231) definition of epidemic curve >>>A graph in which the number of new cases of a disease is plotted against an interval
of time to describe a specific epidemic or outbreak.

232) female 25yo , ask you about breast self examination when should be
done:
-6-7 day after cycle
-5 day befor
-7-10 day after
-14-16 day after
- after 2 day
A in 4th
233) pt underwent colectomy dignose as stage B2:
- NO lymph node involve
-2to4 l.n
- one l.n
A in 4th
234) Ttt for menpusalwomen ,c/o bleeding , not ass with intercourse:
-estrogen
-progesteron
B in 4th
235) Picture ,Child with skin lesion at elbow , seen positive wood lamp:
-fungal
-bacterial
A in 4th
236) Most common problem present in primary care :coryza
237) Firest sign in increase intracraineal pressure:
-vomiting
-nausea
-ipsilateral pupil constrict
-cotralateral pupil constrict
Answer is: decreased level of consciousness in 4th
The full sequence is: decreased level of consciousness, confusion, headache,
projectile vomiting, unequal pupils (anisocoria), and the presence of a pronator
drift or motor weakness
238) Indication for tonsillectomy is:
a. Pharyngeal abscess
b. Sleep apnea
c. Recurrent infection
d. Asymmetric tonsillar hypertrophy
C they said

239) Patient loss his wife in the last 4 months , he looks sad cannot sleep in
the last 2 days, which medication can help him:
e- Lorazepam
f- Diazepam
g- SSRI.
E in 4th they said g
240) Most common organism causing cellulitis in the age 6-24 month
E. Strepto coccus
F. Heamophilus influ
G. Staph
G they said
241) Gastrictomy post-op 1 day. He have temperature 38.8 & pulse 112. What is
the most common cause ?
a. wound infection.
b. inflammatory mediator in the circulation.
c. UTI
d. normal
Category Day Description
Wind POD1-2 the lungs, i.e. pneumonia, aspiration, and pulmonary embolism. Once attributed to atelectasis, but this has
been shown to be inaccurate.[3][4]
Water POD3-5 urinary tract infection, related to indwelling catheter (during surgery or currently i.e. Foley catheter )
Walking (or VEINS, which then sounds like "Weins") POD4-6 deep vein thrombosis or pulmonary embolism
Wound POD5-7 surgical site infection, which in obstetrics or gynaecology, may refer to the uterus.
Wonder drugs or What did we do? POD7+ drug fever, infections related to intravenous lines or reaction to blood products
242) Most common cause of recurrent tonsillitis :
f- Group B streptococcus
g- EBV
h- Bacteriod
i- Rhino virus
j- Parainflunza virus.
F they said
243) Holding breath spell or holding ..which of the following is true
A)mostly occurs between age 5-10
B)increase risk of epilepsy
C)a known precipitant cuz of generalized convulsion
D)diazepam may decrease the attack
e) can occur in absence of emotional upset

244) Yong male with 3 day of dusuria, anal pain , O/E perrectum boggy
mass :
- acute prosatities>>>> in 4th
245) Pt with high total chloestrol 265mg/dl , LDL 150 , triglecride 325 , HDL
100
most single risk factor???
- low LDL
-High LDL
-High HDL
-low HDL
-high total cholesterol
B in 4th
246) patient has history of parotid and salivary gland enlargment
complains of dry eye . mouth and skin ,, lab results HLA-B8 and DR3 ANA
+ve rheumatoid factoe +ve what is the course of treatment
a-physostigmin
b-eye drops with saliva replacemnt
c-NSAID
D-plenty of oral fluid
B in 4th
247) battered women which is true:
a. mostly they come from poor socioeconomic area
b. usually they marry a second violent man
c. mostly they come to the E/R c/o..
d. mostly they think that the husband respond like this because they still
have strong feeling for them
a in 4th
248) most common physiological cause of hypoxemia
- shunt
-Ventilation perfusion mismatch
-hypoventilation
B in 4th
249) arterial bleeding after injury:
- red blood ,continous
-red bright , spruting
-dark blood
B in 4th

250) teacher c/o malise fever , right upper abd tenderness , two student
develop same condition , eye become icterus, best CONFIRM dignose:
-HBA IgG
- HBA IgM
- HBA core AB
B they said
251) Young female always eat fast food , you advice supplement of:
-zinc +vit. C
-vit. C+ folic
-vit.d+ zinc
-folic acid+ Ca
D they said
252) Pt dx to have aortic stenosis ,,, he is a teacher ,, while he was in the class he
fainted,,, what is the cause??
E. Cardiac syncope
F. Hypotention
G. Neurogenic syncope
F they said
253) HBV serological marker (Know what is the 1st marker that rises and what
rises at the window area and what rises after 20 wks )
HBsAg : indicate carrier state.
HBsAb : indicate provide immunity to HBV
HBcAg: associated with core of HBV
HBcAB: during widow period, HBcAb-IgMindicate recent disease

254) unstable angina dx:


- least grade II and new onset less than 2 months ago.
-usually there is an evidence of myocardial ischemia.
-same ttt as stable angina.
-discharge when the chest pain subsides.
255) pregnant in 35 week with mild preeclampsia, presented with BP 150/95 and
edema in lower and upper limbs, how to manage?
-diuretics
-immediate delivery
-maternal and fetal evaluation and hospitalization (correct) in 4th
256)well tolerated in pregnancy: mitral regurg (got 5/5 in ob) in 4th
257) best place to find gonoccal in females:
urethra
rectum
cervix
posterior fornix of vagina
pharynx
258) snoring + tonsillar enlargement:
weight loss,

CPAP,
adenoidectomy (correct) in 4th
259) patient has been wearing contact lenses for the past 10 years, now has
photophobia, what do you recommend?
-take them off at night (correct) in 4th
-saline drops 4 times a day
260) OPTHALMOLOGY 3/3 : all in 4th
1- PICTURE OF AN EYE : NO HX OT DISCHARGE ONLY TEARY EYE AND REDNESS
ETC : DX WAS VIRAL CONJUNCTIVITIS.
2- HTN LADY WITH HIGH LEVEL OF PB HEADACHE , OPTHALMOLOGIC
EXAMINATION SHOWED CUPPING AND EXTRA FININGS WHICH I DONT RECALL IT :
MOST APROPRIAT MANAGEMENT WAS : URGENT REFERRAL TO
OPHTHALMOLOGIST.
3- DRY EYE , PRESCRIPE THE DOSE OF THE LUPRICANT : 1 DROP IN THE LOWER ..ETC.
261) Most important physiological process of hypoxia ventilation/perfusion
mismatch (SURE) in 4th
262) Treatment of bacteroides fragilis :clindamycin (SURE( in 4th
263) all sure in 4th
-Initial Tx of psoriasis with 15% body involvement :
Topical steroid (SURE(
-Asthmatic pt with scales on face and forhead & antecubital fossa Atopic
dermatitis (SURE(
-Ricotan tx for ace can cause :birth defect (SURE(
-Main symptom of AOM is :pain (SURE(
-Nasal obstruction in one nostril ttt: Steroid (SURE(
-Most common cause of epistaxis in children:self induced trauma (SURE(
-child with baking cough,stridor,and mild fever 38 Dx: croup (SURE(
-child with cough,runny nose and fever,O/E:tonsillitis ttt: paracetamol and throat swab (SURE(
-child with blab la bla x-ray(steeple sign ): croup (SURE(
-Mitral stenosis murmur : diastolic low pitched,rumbling ( SURE(
-child with URTI then arthralagia and fever Dx: rheumatic fever(SURE(
-OCP can cause changes in : cervical mucosa (SURE(
-Congenital heart disease with greatest risk of endocarditis: TOF (SURE(
-Obese child with BMI=30,blab la bla lifestyle change (SURE(
264) red eye with watery discharge:
local antihistamine

steroids
antibiotics
A they said
265) In which medical condition Methylergometrine (methergine)
Which use in postpartum hemorrhage is contraindication
1-maternal asthma
2-maternal hypertension
266) which one non-pharmacological is the most appropriate in hypertension
1-whight loss
2-low-deit salt
3- decrease alcohol
4- stop smoking
1 they said >>1 5% wt loss ass with a decrease of diastolic pressure of around 10
267) pt with did colectomy after colon cancer , now lymph node showing
micro????((mean met to lymph ))
1-it is sensitive to chemotherapy >>>>> in 4th
268) A 34 year old lady presented with pelvic pain and
menorrhagia. There is history of infertility. On
examinations the uterus was of normal size & retroverted.
She had multiple small tender nodules palpable in the
uterosacral ligament. The most likely diagnsosis is:
a) endomytritis
b) Endometriosis ( true ) in 4th
c) Adenomyosis
d) PID
269) In which medical condition Methylergometrine (methergine) Which use in postpartum hemorrhage is contraindication
1-maternal asthma
2-maternal hypertension
2 they said
270) In cystic fibrosis the genetic defect in
1-short arm of human chromosome 7.
2-long arm of human chromosome 7.
3-short arm of human chromosome 17
2 they said
271) Rebound phenomena
It cause by over use of vasoconstriction medication>>>in 4th

272) Pt with sudden cardiac arrest the ECG showed no electrical activities with
oscillation of QRS with different shapes. The underlying process is:
a. Atrial dysfunction
b. Ventricular dysfunction
c. Toxic ingestion
d. Metabolic cause
B they said >>b electrical alternance a sign of LVF
273) patient having chest pain radiating to the back, decrease blood pressure in
left arm and absent left femoral pulse with left sided pleural effusion on CXR, left
ventricular hypertrophy on ECG, most proper investigation to dx:
1-aortic angiogram
2-amylase level
3-cbc
4-echo
1 they said
274) male old patient has S&S of facial palsy ( LMNL) ; which of
the following correct about it ;
A- almost most of the cases start to improve in 2ed weeks
b- it need ttt by antibiotic and anti viral
c- contraindicated to give corticosteroid
d- usually about 25 % of the cases has permanent affection
A they said
275) Primigravida with whitish discharge the microscopic finding showed
pseudohyphae the treatment is:
a. Meconazole cream applied locally
b. Tetracycline
c. Metronidazole
d. Cephtriaxone
A they said
276) pt with 18 months amenorrhea,high FSH,divorced:
a-pregnancy
b-premature ovarian failure
c- hypothalamic amenorrhea
d- pituitary microadenoma
B they said
277) The heart increase its blood supply by
- Pulmonary resistnace

- Dilate coronary artery


- Constrict aortic artery
B they said
278) Na high, K low, HCo3 high:
primary hyperaldosteronism,
addison
pheochromocytoma
A they said
279) HIV pt. with skin lesion show (spindle cells) Diagnosis:
- Kaposi sarcoma>>>> in 4th
280) Young patient with pharyngitis, inflammation of oral mucosa and lips that
has whitish cover and erythmatous base, febrile, splenomegaly. Dx:
a. Scarlet fever
. EBV (my answer) in 4th
. HZV
281) if the likelihood ratio .3 what does that mean ?
1- No change
2- Small increase
3- Large decrease
4- Moderate decrease
4 in 4th
No choose of small decrease
282) Pap smear ::
One collection from os of cervix
3 collection from the endocervical canal
One collection from vagina
283) This picture ,,, fluoresce a coral red colorin wood lamp << what is the

diagnosis:
Erythrasma.
Candida
Psoriasis
A in 4th
284) hx of long standing abdominal pain improve with peptic ulcer medication,,
present with abdominal pain,distention, forcfull vomiting, emesis contain

morning food .. diagnosis:


Gastroparesis
Gastric outlet obstruction
Dialted cardia
Esophageal reflux
B they said
285) berberi = B1 deficiency >>> in 4th
286) Case scenario of pellagra (diarrhea, dermatitis, and dementia, glossitis) >>> in 4th
287) True regarding perths disease
Affect girl > boy
Common age 11-16
Always unilateral
Painless
d they said>> Perths disease affect boy more than girl-age 5-10-maybe unilateral or bilateral-it could be painless limping or
painful limping
288) Pt with adult respiratory distress syndrome.. he got tension pneomothorax..
what is the
propable cause:
a-severe lung injury
b-Negative pressure
c-central venous line
A they said
289) Which breast disease is Bilateral:>>Lobular carcinoma in 4th
290) pregnant never did check up before , her baby born with
hepatosplenomegaly and jaundice :
a-Rubella
b-CMV
c-HSV
d-HIV
B they said
291) 4years old child what can he do
1.Copy square and triangle
2.Speak in sentences
1 I said
292) baby can sit without support, walk by holding furniture. Pincer grasp, pull to
stand how old is he

a.8 months
b.10 months
c.12 month
d.18 month
b I said >>>9-10 months
293) young patient complain of watery diarrhea, abdominal pain.. with a
previous hx of mucus diarrhea. Symptom improve when sleep
Chrons
UC
IBS
C they said
294) ttt of foliculitis
a- oral steroid
b- topical steroid
c -oral antibiotics
c in sever, for mild use systemiv ab >> large lesion incised, drained & cultured to rule out MRSA
295) Old patient with bilateral enlarged knee , no history of trauma , no tenderness
,
normal ESR and C-reactive proteins . the diagnosis is
a. Osteoarthritis
b. Gout
c. Infectous arthritis
A they said
296) What is the initial management for a middle age patient newly diagnosed
knee osteoarthritis.
a. Intra-articular corticosteroid.
b. Reduce weight.
c. Exercise.
d. Strengthening of quadriceps muscle.
B they said
297) what proven to reduce incidence of cancer ?
1-fiber
2-vit c
3- vit D
4- ca
5- folic acid

3 they said
298) What is true about Propylthiourocil :
block thyroid hormones.>>>>> in 4th
299) Pt walking fot relativly long time on ice whene she was in vacation
(somewhere in cold area) her feet is pale with marked decrease in pain
sensation but the pulse is palpable over dorsalis pedis what is the appropriate
thing to do:
a-immedate heat with warm air
b-put her feet in worm water.
B they said
300) 18 years old boy complaining of fever 38c, flank pain, pain during
urination(dysuria) for 4 days,urine analysis showed WBC 50 to 60,,,your action
?
a) ciprofolxacine 500 for 2 days and to came back to clinic
b)pinicilin for two weeks & to be seen in the clinic for reussurance.
c)admition to hospital and iv antibiotics
C they said
301) patient had burned by hotty oil in the right side of his arm and leg,came
to you in clinic. So you will refferd him to burn clinician specialist if?
a)10 cm painful area with no blusters
b)5 cm painful area with blusters
c)5 cm paineless area with no blusters (third degree ,full thickness)
C they said
302) What is true about Peritonitis :
a. chemical irritation can cause it.
b. Associated with abdominal rigidity which increase as the Paralytic illeus
develops.
A they said
303) patient with complain of calf tender and swellen,,diagnosed to have
DVT,,what is the rule of LOW MOLECULAR WIEGH HEPARIN in DVT treatment as
comparing to unfactionated heparin?
a) LMWH is less effictive
b)LMWH is prone to more bleeding
c)LMWH is safe and no need to regular monitor the PTT.
C they said
304) 6 month old came with sign and symptom or RD " fever, tachyepnia,
intercostals ressesion, expiratory wheez, nazal flare".. best intial management :

Oxygen
Erythromycin
Bronchodilator
A they said
305) ong scenario abuot pt having epigastric pain radiate to the back increase
with lying and decrease when standind ass fever tachycardia. It is typical
with acute pancreatitis .. what is the next diagnostic step:
a-abdominal CT
b- abdominal Xray
c-ERCP
d-seum amilase and lipase
d they said
306) m baby with mild viral diarrhea , ttt by ORS as
a-100ml/kg for 4 hour then 50 ml/kg /day after
b-50>>>>>>>>>>>>>>>>.50>>>>>>>>>>
c-100>>>>>>>>>>>>>>>>100>>>>>>>>
d-50>>>>>>>>>>>>>>>>>100>>>>>>
D mai said
307) stap wound in triceps.. days late green wound discharge.. on wound
examination no resistance ,, what is the organism:
-streptococcal
-gangrene fournier
-Clostredia gangrene
A they said
308) Facial nerve when it exits the tempromandibular joint and enter parotid
gland it passes:
a) Superficial to retromandibular vein and ext. carotid artery
b) deep to ex. Carotid
c) deep to R vein
d) between retrmandibular vein and external carotid artery
D they said>>> l I read deep to parotid gland lateral to external carotid a nd retromandibular vein:)
309) pt has Lt lower Abdominal pain , Fever , constipation CT reveals thickened
loop and little perianal fat , whats appropriate to do:
1- start AB
2- call the surgeon for immediate OP
3-give laxative
4- barium enema

A they said mai " Diverticulitis "


310) battered women which is true:
a. mostly they come from poor socioeconomic area
b. usually they marry a second violent man
c. mostly they come to the E/R c/o..
d. mostly they think that the husband respond like this because they still
have strong feeling for them
B I said
A battered woman is a woman who has experienced at least two complete battering cycles as described in dating and
domestic violence.
Battered women stay in these dangerous relationships for a variety of reasons including:

still being positively reinforced by the "honeymoon" phase of the battering cycle

economic dependence upon the batterer

belief that they can keep the peace

fear of danger if she were to leave

threats made by the batterer to hurt her or her children if she left

loss of self-esteem

depression or loss of psychological energy necessary to leave

According to Walker's The Battered Woman Syndrome (p. 95-97, 1984), there are four general characteristics of the
syndrome:
1.

The woman believes that the violence was or is her fault.

2.

The woman has an inability to place responsibility for the violence elsewhere.

3.

The woman fears for her life and/or her children's lives.

4.

The woman has an irrational belief that the abuser is omnipresent and omniscient.

More recently, Battered Woman Syndrome has also been associated with post-traumatic stress disorder in that domestic
violence involves exposure to severe trauma and so the reactions of a battered woman may be due to flashbacks or other
intrusive experiences from previous traumatic events so that the woman believes that she is in danger, even if she is not.
311) Pt with high total chloestrol 265mg/dl , LDL 150 , triglecride 325 , HDL
100
most single risk factor???
- low LDL
-High LDL
-High HDL
-low HDL
-high total cholesterol
B they said
312) Pt dx to have aortic stenosis ,,, he is a teacher ,, while he was in the class he
fainted,,, what is the cause??
E. Cardiac syncope
F. Hypotention
G. Neurogenic syncope
F they said
313) reatment of mania that does not cause hepatotoxicity
a. Lithium b. carbamazepine c. valporic acid d. lamotrigine
A they said
314) unstable angina dx:
- least grade II and new onset less than 2 months ago.
-usually there is an evidence of myocardial ischemia.
-same ttt as stable angina.
-discharge when the chest pain subsides.
B i said
315) 56 y old present with vasomotor rhinitis
a. Local anti histamine
b. Local decongestion
c. Local steroid
d. Systemic antibiotic
B I said

316) Young female always eat fast food , you advice supplement of:
-zinc +vit. C
-vit. C+ folic
-vit.d+ zinc
-folic acid+ Ca
D they said
317) pt with BP of 180/140 you want to lower the DyastolicBP(which is true ) :
a- 110-100 in 12 hrs
b-110-100 in 1-2 days
c- 90-80 in 12 hrs
d- 90-80 in 1-2 days
C I said >>>Several classes of antihypertensive agents are recommended, with the choice depending on the etiology of the
hypertensive crisis, the severity of the elevation in blood pressure, and the usual blood pressure of the patient before the
hypertensive crisis. In most cases, the administration of an intravenous sodium nitroprusside injection, which has an almost
immediate antihypertensive effect, is suitable (but in many cases not readily available). In less urgent cases, oral agents
like captopril, clonidine, labetalol, or prazosin can be used, but all have a delayed onset of action (by several minutes)
compared to sodium nitroprusside.
It is also important that the blood pressure be lowered smoothly, not too abruptly. The initial goal in hypertensive
emergencies is to reduce the pressure by no more than 25% (within minutes to 1 or 2 hours), and then toward a level of
160/100 mm Hg within a total of 26 hours. Excessive reduction in blood pressure can precipitate coronary, cerebral, or
renal ischemia and, possibly, infarction.[11]
The diagnosis of a hypertensive emergency is not based solely on an absolute level of blood pressure, but also on the
typical blood pressure level of the patient before the hypertensive crisis occurs. Individuals with a history
of chronichypertension may not tolerate a "normal" blood pressure.
318) COPD coughing greenish sputum, whats the organism?
staph aureus
strep pneumonia
mycoplasma
chlamydia
h.influenza
B they said >>>Then hib then moraxella catarahlis

319) Most common cause of postmenopausal benign bleeding:


a- cervical polyp
b- cervical ca
c- vaginitis
d- endometrial hyperplasia
>>>Postmenopausal bleeding can originate in different parts of the reproductive system. Bleeding from the vagina may
occur because when estrogen secretion stops, the vagina dries out and can diminish (atrophy). This is the most common
cause of bleeding from the lower reproductive tract.
Lesions and cracks on the vulva may also bleed. Sometimes bleeding occurs after intercourse. Bleeding can occur with or
without an associated infection.
Bleeding from the upper reproductive system can be caused by:

hormone replacements

endometrial cancer

endometrial polyps

cervical cancer

cervical lesions

uterine tumors

ovarian cancer

estrogen-secreting tumors in other parts of the body

The most common cause of postmenopausal bleeding is HRT. The estrogen in the replacement therapy eases the symptoms
of menopause (like hot flashes), and decreases the risk of osteoporosis. Sometimes this supplemental estrogen stimulates
the uterine lining to grow. When the lining is shed, post-menopausal bleeding occurs. Most women on HRT usually take the
hormone progesterone with the estrogen, and may have monthly withdrawal bleeding. This is a normal side effect.

About 5-10% of postmenopausal bleeding is due to endometrial cancer or its precursors. Uterine hyperplasia, the abnormal
growth of uterine cells, can be a precursor to cancer.
320) red eye with watery discharge:
local antihistamine
steroids
antibiotics
A they said
321) Best view to see the rib fracture
1-posterior-anterior x-ray
2- anterior-posterior x-ray
The antero-posterior chest radiograph will identify most significant chest wall injuries, but will not identify all rib fractures.
Lateral or anterior rib fractures will often be missed on the initial plain film. However, since the management of rib fractures
is determined by their clinical significance rather than by their number or position, dedicated rib views are never indicated.
For adult blunt trauma patients, a haemothorax, pneumothorax or pulmonary contusion seen on chest X-ray will almost
always be associated with a rib fractures, whether or not identified clinically or by X-ray. In paediatric patients the ribs are
more pliable and less likely to fracture, although there will still be significant contusion of chest wall structures.
322) Child sqagueled for elective surgery his wieght is 22 kg,, what is the fluid
deficit to give?
a) 37ml/h
b) 65ml/h
c) 90ml/h
d) 88ml/h
B they said>>>>
323) A 37-year-old man was referred for evaluation of Renal and metabolic Functions . Laboratory evaluation revealed a
serum potassium level of 3.3 mmol per liter,

a bicarbonate level of 16 mmol per liter,


a calcium level of 9.3 mg per deciliter (2.3 mmol per liter),
a phosphate level of 2.1 mg per deciliter (0.7 mmol per liter),
a creatinine level of 3.0 mg per deciliter (265 mol per liter),
a parathyroid hormone level of 62 pg per milliliter, and an estimated glomerular filtration rate of 25 ml per minute per 1.73
m2 of body-surface area
What is the most likely diagnosis?
Renal tubular acidosis
Primary hypoparathyroidism
Familial hypocalciuric hypercalcemia
Salicylate overdose
Pagets disease
A they said
324) newborn with fracture mid clavicle what is true:
a. Most cases cause serious complication
b. Arm sling or figure 8 sling used
c. Most patient heal without complications
C they said
325) Pt with Ami nd PVC ttt: A/amoidarone. B/lidocaine. C/no ttt
B they said

326) Which personality disorder is associated with inflexibility and perfectionism:


a. Narcissistic personality disorder
b. Borderline personality disorder
c. Obsessive compulsive personality disorder
d. Histrionic personality disorder
C they said
327) 41- 13 years old child with typical history of nephritic syndrome ( present
with an urea , cola color urine , edema , HTN ) what is the next step to DX .
a-renal function test
b-urine sediments microscope
c-US
d-renal biobsy
B they said
328) Lichen planus most common site ?
Scalp
Knee
Buttocks
Mouth
D they said
329) What is the drug that make cholecystitis more worse :
a.morphine
b.nalaxone
c.phoso
d.merpidine
A they said

330) female presented with Amenorrhea - exesive excercise - hairsotism - diagnosis:


a- ovarian faliure
b- adrenal problem
c- hypothalams
d- pitutary adenoma
C they said
331) 17 year pt with dyspnea Po2 , PCO2 ,Xray normal PH increase so dd is:
a. acute attack of asthma
b. P E
c. pneumonia
d. pnemothrax
A they said
332) ttt of open tibial fracture:
a)cephazolin
b)cephazolin+gentamycin
c)gentamicin
d)cephazolin+gentamicin+metronidazole
D they said>>>> Intravenous antibiotics are administered promptly. First-generation cephalosporins (Gram-positive
coverage) such as cephalothin (1-2 g q6-8h) suffice for Gustilo-Anderson type I fractures. An aminoglycoside (Gramnegative coverage) such as gentamycin (120 mg q12h; 240 mg/d) is added for types II and III injuries. Additionally,
metronidazole (500 mg q12h) or penicillin (1.2 g q6h) can be added for coverage against anaerobes. Tetanus prophylaxis
should be instituted. Antibiotics generally are continued for 72 hours following wound closure

333) which of the following found to reduce the risk of postherpetic neuralgia:
a)corticosteroids only
b)corticosteroids+valacyclovir
c)valacyclovir only
C they said
334) 32 yrs old man brought after he get fall from motorbike and he did not use helmet, 6 minute before arrive to the
hospital become unconscious, respond to painful stimuli, LT eye deviated to LT side and fixed dilated pupil, what is the
cause? a-cerebral hemorrhage b-occipito cerebral hemorrhage c-subdural hemorrhage d-...............
C they said
335) in Henoch-Schonlein Purpura which one is considered pathological
a. gross hematurea
b. microscopic hematurea
c. rashes
C they said >>>> The following may be noted in the history of patients with Henoch-Schnlein purpura (HSP):

The prodrome is associated with the following: Headache


Anorexia
Fever

After the prodrome, a rash, abdominal pain, peripheral edema, vomiting and/or arthritis develop. The rash appears in
100% of patients and is the presenting feature in 50%.
The distribution usually depends on parts of the body, including the lower trunk, lower extremities, buttocks and perineum.
The rash typically appears in crops with new crops appearing in waves.
Eruptions usually last an average of 3 weeks.
As many as 85% of patients will have GI symptoms, including abdominal pain, nausea, and vomiting.
The most common symptom is colicky abdominal pain.
Joint involvement is present in 75% of reported patients with Henoch-Schnlein purpura and the presenting sign in

approximately 25%.
The large joints (eg, knees and ankles) are most commonly involved, with pain and edema being the only symptoms. The
arthritis resolves completely over several days without permanent articular damage.
Renal involvement is present in 30-50% of patients and may persist as long as 6 months after the onset of the rash.
Renal involvement manifests in a range from mild hematuria or proteinuria to oliguria and renal failure.
Permanent renal impairment is seen in 20% of patients who have nephrotic or nephritic syndrome; however, this turns out
to be less than 0.1 % of all patients diagnosed with Henoch-Schnlein purpura.
336) Female patient presented with tender red swelling in the axilla with history of repeated black head and large pore skin
in same area: ttt is
a. Immidate surgery
b. Topical antibiotic
c. Cold compressor
d. Oral antibiotic
A they said>>>> infected sebaceous cyst treatment is surgical excision...a
337) Pt presented with acute MI after 24hrs he developed ECG changes shown below,what is z ttt; A/lidocaine B/ lidocaine or
amoidarone C/amoidarone

C they said>>>> This is wide complex VT. if present after 24hr we give amoidarone if less than 24hr we give lidocaine or
amoidarone
338) regarding hypokalemia can cause all true EXCEPT :
a- mental depresson
b- musle paralysis
c- cause hyperosmolar coma

C they said>>> hypokalemia can cause :


T wave inversion
Myopathy
Diabetes insipidus, nephrogenic
Cardiac arrhythmia
Urine pH low
U waves tall
Atrioventricular node conduction block
Alkalosis, metabolic
Rhabdomyolysis
Bicarbonate levels raised (plasma)
Multifocal atrial tachycardia
QRS prolongation
ST segment depression
Ileus
Cardiac arrest
Acute confusional state
QT lengthening
T waves flat
Constipation
339) Secondary dysmenorrhea is:
a) rare due to anovulation.
b) due to gonadal agenesis
c) always pathological
d) part of sheahan syndrome
C they said
340) Patient with Sever hypothyriodism and hyponatremia (108= Na ), high TSH and not respond to painfull stimuli, how
would you treat him :
a. Oral intubation , Thyroid replacemet , Steroid and 3% Na
b. Same above but Without steroid
c. Thyroid and fluid replacements only
d. Thyroid and fluid and%3 Na
A they said

341) CCB drugs like verapmil , dilitazem , nifedipine are effective except:
a- prinzmetal angina
b- HTN
c- atrial tachycardia
d- ventricular tachycardia
e- effort angina
D they said
342) all are true about the best positon in hearing the murmurs ,except :
a- supine : venous hum
b- sitting : AR
c- sitting : pericardial rub
d- supine : innocent out flow obstruction
e- Lt latral in : MS
A They said
343) all are speech disorders except:
a) Stuttering
b) Mumping
c) Cluttering
d) Palilia
B They said
344) What is true about alpha blocker:
a.Causes hypertension.
b.Worsen benign prostatic hyperplasia.
c.Cause tachycardia.
C They said
345) All can cause gastric ulcer except:
a- Tricyclic antidepressant.
b- Delay gastric emptying.
c- Sepsis.
d- Salicylates.
e- Gastric outlet incompetent.

A They said
346) The investigation of high sensitivity and specificity of urolithiasis :
1. IVP
2. X RAY abdomen
3. US
4. MRI
5. nuclear scan[/LEFT
A They said
347) pt with hypertension by fundoscopy there is cupping of optic disc , what will you suspect :
1.hypertensive retinopathy
2.open angle glaucoma
3.angle clousure glucoma
B They said
348) Old man with cognitive deficit what we will screen?
a) Iq test
b) Involuntary movement test
c) MEMORY score test
d) Hearing test
C They said
349) Treatment of pt with yellowish vaginal discharge and itchy by swab and cultur it is Trichomonas vaginaliswhich of the
following is correct :
1. start treatment with metronidazol
2. start treatment with clindamycin
3. no need to treat husband
4. vaginal swab culture after 2 weeks
A They said
350) which of the following causes the highest maternal mortality in pregnancy
a. toxoplasma
b. hyperbilirubenia

c. pheochromocytoma
d.rubella
A They said
351) Rt lunge anatomy ?
a-2pulmonary vanes
b-2 fisser
c-8segments
B They said>>> Fissures are
Major Fissure (aka oblique fissure)
minor fissure (horizontal fissure)
there are 10 segments & 4 pulmonary viens
am I right ??
352) 35 year old smoker , on examination shown white patch on the tongue, what is the management?
a) Antibiotics
b) No treatment
c) Close observation
d) Excision biopsy
D they said
353) 23 years old lady with one month history of nasal discharge & nasal obstruction, she complained of pain on the
face, throbbing in nature, referred to the supraorbital area, worsen by head movement, walking,& stopping.
On examination , tender antrum with failure of transillumination ( not clear ), the most likely the diagnosis is:
a) frontal sinusitis
b) maxillary sinusitis (not sure)
c) dental abscess
d) chronic atrophic rhinitis
e) chronic sinusitis
a they said
354) Patient was presented by ear pain , red tympanic membrane , apparent vessels , with limited mobility of the
tympanic membrane , what the most likely diagnosis :
a) Acute otitis media

b) Tympanic cellulitis.
c) Mastoditis
a they said
355) 50 y with uncontrolled diabetes ,complain of black to brown nasal discharge. So diagnoses is
a- mycomyosis
b- aspirglosis
c-foreign body
a they said
356) Pt has a scaly hypopigmented macules on the chest and arms
They seem even lighter under the sunlight,,, what is the ttt?
-Topical steroid
-Na selinum
-Topical antibiotics
-Oral antibiotics
B they said>>> pityriasis versicolor
357) Which most common condition associated with endocarditis
a-VSD
b-ASD
c-PDA
d-TOF
a they said
358) Which heart condition is tolerable during pregnancy:
a. Eisenmenger syndrome
b. Aortic stenosis
c. Severe mitral regurge
d. Dilated cardiomyopathy with EF 20%
e. Mitral stenosis and the mitral area is 1 cm (or mm).
C they said>> Young women with uncomplicated secundum-type atrial septal defect (ASD) or isolated ventricular septal
defect (VSD) usually tolerate pregnancy well. Patent ductus arteriosus (PDA) is not associated with an additional maternal
risk for>> Chronic mitral regurgitation most commonly is the result of myxomatous degeneration or rheumatic heart

disease and usually is well tolerated during pregnancy. However, new-onset atrial fibrillation or severe hypertension can
precipitate>>> Women with severe mitral regurgitation and signs of cardiac decompensation before pregnancy are advised
to undergo operative repair before conception. Mitral valve prolapse

359) Patient presented to you complaining of left submandibular pain and swelling when eating. O/E, there is
enlarged submandibular gland, firm. What is the most likely Dx?
a) Mumps
b) Sjogrens syndrome
c) Hodgkins lymphoma
d) Salivary gland calculi
D they said
360) Infant born with hemangioma on the rt eyelid what is appropriate time to operate to prevent amylopia:
a.one day
b.1 weak
c.3monht
d.9month
B they said
361) female he son is 6month age came to the clinic with tremor palpitation d.
1.post partum thyroiditis
2.hyperthyrodism
3.subacute thyroiditis
hypothirodism
A they said
362) 12 yr old girl with malaise, fatigue, sore throat & fever. On examination: Petechial rash on palate, large tonsils
with follicles, cervical lymphadenopathy & hepatosplenomegaly. All are complications EXCEPT:
a) Aplastic anemia
b) Encephalitis
c) Transverse myelitis
d) Splenic rupture
e) Chronic active hepatitis

E they said>>> CNS complications include meningitis, encephalitis, hemiplegia, Guillain-Barr syndrome, and transverse
myelitis. EBV infection has also been proposed as a risk factor for the development of multiple sclerosis (MS),[39] but this
has not been confirmed.
Hematologic: Hemolytic anemia (direct Coombs test is positive) and various cytopenias, and bleeding (caused by
thrombocytopenia) can occur.[14]
Mild jaundice
Hepatitis with EBV is rare.
Upper airway obstruction from tonsillar hypertrophy is rare.
Fulminant disease course of immunocompromised patients is rare.
Splenic rupture is rare.
Myocarditis and pericarditis are rare
363) Medication increase reflux esophagitis
a- Theophylline ?
b-ranitidine
c-plasil
d-ampicillin
A they said
364) 31 year old Women with cyclic bilateral modularity in her breast since 6 months on examination there is 3 cm tender
mobile mass wt u will do next:
a-FNA with cytology
b-mammogram
c- biopsy
d- follow up for next cycle
e-observation
A they said>>> The young woman with multiple nodules and diffuse thickening consistent with fibrocystic change has
difficult breasts to evaluate. Reexamination at different times in the menstrual cycle is often informative and reassuring
when no dominant nodule emerges. A persistent solitary or dominant nodule requires biopsy.>>> Evaluation of a dominant
breast mass can most efficiently begin with either FNAB or ultrasonography. If ultrasonography shows a simple cyst, no
further workup is warranted. A complex cyst of solid lump should be followed by FNAB, core-needle biopsy, or excisional
biopsy. If the evaluation begins with FNAB and there is a>>> The woman with painful breasts associated with fibrocystic

change can be reassured that the discomfort is not a sign


P.789
of cancer and that symptoms usually improve with the cyclical decrease in hormonal stimulation. Moreover, it is important
to emphasize that the finding
365) 2 y/o child presented with painful swelling on the dorsum of both hands and feet,, he was jaundiced with T.billi 3
D.billi .9 ,, HGB 9 and retics 7,, what u will do as ongoing management
a. Steroid
b. NSAID
c. penicillin and immunization
d. paracetmol
C they said
366) In paracetamol toxicity:
a. Pencelinemia
b. N-acetylcysteine
c. K intake
d. Dexoamin
B they said
367) pt have vericose vein in her last pregnancy which is not changed , she wear stocks and elevate her legs she asked
about furthur cosmetic options you will told her
A. nothing can be done more
B-stripping will make it worse
C-coagulation therapy
D. saphenous vein laser treatment
D they said
368) Child presented with gum and nose bleeding and bruising all over the body after an episode of URTI. Dx:
a. Henoch Scholein Purpura
b. Idiopathic thrombocytopenic purpura

c. Vitamin K deficiency
d. Hemophilia
A they said
369) Lady want to come pregnant and want to take varcilla vaccine, what you will tell her
A- varcilla vaccine will not protect pregnant lady
b- she should wait 1 - 3 months before coming pregnant
c- it is a live attenuated bacterial
B they said
370) 45years old lady presents with bloody nipple discharge. Most likely Dx:
a. Breast ca.
b. Fibroadenoma
c. Ductal Papilloma.
d. Ductectasia.
C they said
371) case with positive Gowers' sign, which area affected
a-Dorsal column
b-Cerebellum
A they said

372) X-ray pelvis of what is diagnosis:


a.osteomalacia
b.osteoprosis
c.rickets disease
d.paget disease

D they said
373) CSF in Aseptic meningitis
a- Low Protein
b- High glucose
c- Neutrophils
d- Lymphocytes
e- Esinophils
D they said
374) Female pt for evaluation she had 3 prevuse termination by D&C, OE she was normal dx
a- asherman syndrome

b- shehan syndrome
c- kalman syndrom
d- polycystic ovarian syndrome
A they said
375) On examination of newborn the skin show papules or (pastules) over erythema base:
1.transient neonatal pustular melanosis
2.erythema toxicum neonatorum
2 they said
376) Young pt with hx of cough, chest pain, fever CXR showed RT lower lobe infiltrate:
a. Amoxicillin
b. Ceferuxim
c. Emipenim
d. Ciprofloxacin
A they said
377) middle aged woman with multiple sclerosis , complaining of urinary incontinence..she doesn't feel the urge to empty
her bladder but urine incontinence occurs..??
a-Reflex incontinence
b-Stress incontinence
c-Overflow incontinence
d-Urge incontinence
A they said
378) Young patient with decreased hearing and family history of hearing loss, ear examination was normal Rene and Weber
test revealed that bone conduction is more than air conduction, what would you do?
a-Tell him it's only temporary and it will go back to normal.
b-Tell him there is no treatment for his condition.
c-Refer to audiometry.
d-Refer to otolaryngeologist

D they said
379) the separation of chromatid occur in late phae of :
a- anaphase
b- metaphase
c- telophase
B they said
380) child with anuresis, what to do
a-CBC
b-kidney function test
c-urine culture
d- renal biopsy
C they said
381) Drug that will delay need of surgery in AR:
a. digoxin
b. verapamil
c. nefidipin
d. enalpril
C they said
382) A child swallowed his relative's medication. What is the best way of gastric decontamination?
a) Gastric lavage
b) Total bowel irrigation (whole bowel wash)
c) Syrup ipecac
d) Activated charcoal
A they said
383) Pt. after stroke , he lost his smell sensation.. Which part is affected
a- Frontal
b- Temporal

c- Occipital
d- barietal
B they said
384) what the effect of niacin if taken :
a.decrease uric acid .
b.hypoglycemia
c.increase LDL
d.increase HDL
e.increaase triglyceride
D they said
385) what is the most common cause of death in patients with Ludwig's angina?
a-sepsis
b- asphyxiation
c-rupture of the wall
B they said>>> Ludwig's angina, otherwise known as angina ludovici, is a serious, potentially life-threatening cellulitis[1]
[dead link], or connective tissue infection, of the floor of the mouth, usually occurring in adults with concomitant dental
infections and if left untreated, may obstruct the airways, necessitating tracheotomy. It is named after the German
physician, Wilhelm Friedrich von Ludwig who first described this condition in 1836.[2][3] Other names include "angina
Maligna" and "Morbus Strangularis".
386) Diagnosting peritoneal lavage (DPL) positive when:
a-RBC 1000
b-WBC 50
c-2ml at aspiration
d-blood in chest tube
e-2ml in pregnancy
>>if any of the following are found then the DPL is positive of trauma and operative exploration is warranted:[7]
10 cc/blood
100,000 RBCs/mm3

500 WBCs/mm3
Presence of bile, bacteria or food particles
387) kwashiorkor :
a-high protein and low carbohydrate
b-high protein and high carbohydrate
c- low protein and high carb
d-low preotein and low carb
C they said>>>it should be low proteins n normal carbs
388) pt with meniere disease advised to take
low salt no caffiene
low salt high caffiene
high salt no caffiene
high salt low caffeine
A they said
389) 65 y/o pt. presented with hepatosplenomegaly and
lymphadenopathy...bone marrow bx confirm dx of CLL,, the pt gave hx
of breast cancer 5 yrs ago and was treated with radiotherapy since then
,, the pt is also smoker what is greatest risk for developing CLL??
a. hx of radiation
b. smoking
c. previous cancer
d. age
D they said>>>> The cause of CLL is unknown. There is no evidence showing that exposure to radiation, chemicals, or
chemotherapy increases a person's risk of developing CLL. The following factors may raise a person's risk of developing
CLL>>> Family history.AgeGender. Men develop CLL more often than women.
Ethnicity>>> Age. CLL is most common in older adults, is rare in young adults, and hardly ever develops in children. About
90% of people diagnosed with CLL are older than 50. The average age at diagnosis is close to 70 years

390) pt. with primary biliary chirhosis wich drug helps the histopathology of the liver:
- Steroid
- Interferon
- Ursodiol.
C they said>>> Ursodeoxycholic acid (Ursodiol) is the most frequently used treatment. This helps reduce the cholestasis
and improves blood test results (liver function tests). It has a minimal effect on symptoms and whether it improves
prognosis is controversial.
391) Patient with continous seizures for 35 min. despite taking 20 mg Iv diazepam..what to do??
a- give 40 mg IV diazepam
b- give IV phenytoin
c- give IV Phenobarbital
B they said
392) Patient with retrosternal chest pain, barium swallow show
corkscrew appearance:
a. Achalasia
b. Esophagitis
c. GERD
d. Diffuse esophageal spasm
D they said
393) Patient with Hx of severe hypertension, normal
creatinine, 4g protein 24 hrs. right kidney 16cm & left
kidney 7cm with... arteriogram show left renal artery
stenosis. Next investigation:
a. arteriogram
b. biopsy
C. CT angio
d. Bilateral renal vein determination
C they said

394) Following evacuation of a molar pregnancy, B-hCG titers will fall to undetectable levels in about 90% of
patient within:
a) 2 weeks
b) 4 weeks
c) 8 weeks
d) 10 weeks
e) 12-16 weeks
E they said>>> its usually bet. 4-6 months, upto 6 mon
395) what is the fluid recommended for child 9 months old with 10kg:
a-900
b-1000
c-1200
B they said
396) case scenario ptn in labor, baby in late deceleration, what u will do in this case :
a. change position & give O2.
b. give Mg sulfate.
c. give oxytocin
A they said
397) The class of antibody responsible for hemolytic disease of the newborn is:
a) IgA
b) IgG
c) IgM
d) IgE
e) IgD
B they said
398) Anteriolatral insertion of placenta not reach by extended PV EXAM :
a) Low laying placenta
b) Normal
c) Marginal

B they said
399) Elderly patient with dementia and change in his behavior (many things including agitations ) which lobe in
brain affected :
a) Frontal
b) Occipital
c) Temporal
d) Partial
e) Cerebeller
A they said
400) After infarction, the patient become disinhibited, angrier & restless, The area responsible which is affected:
a) Premotor area
b) Temporal area
c) Pre- frontal area
C they said
401) Typical picture of oculomotor nerve palsy: stroke with loss of smell, which lobe is affected?
a) Frontal
b) Parital
c) Occipital
d) Temporal
D they said
402) transmistion of vlavavrus>>>> THE RODNT BIG AND BIRD>>>> t's flava virus and it's transmitted by mosqitoes or
ticks....one of the hemorrhagic fevers
403) Patient came with HTN, KUB shows small left kidney, arteriography shows renal artery stenosis, what is the
next investigation:
a) Renal biopsy
b) Renal CT scan
c) Renal barium
d) Retrograde pyelography

B they said
404) 8 weeks Primigravida came to you with nausea & vomiting, choose the statement that guide youto
hyperemmesis gravidarm:
a) ketonia
b) ECG evidence of hypokalemia
c) Metabolic acidosis
d) Elevated liver enzyme
e) Jaundice
A they said
405) Old male c/o right hip pain on walking the pain increased at the end of day when he wake up in morning he
complaining of joint pain and stiffness
a) Osteoarthritis
b) Ostiomylitis
c) Osteoprosis
A they said
406) What is the deficient vitamin in infantile beri beri :
a) Vitamin B1 thiamine
b) Vitamin C
c) Vitamin E
d) Niacine
A they said
407) ASPIRIN OVER DOSE PT HAD SIZERE PALPITATION TACHYCARDID LOW URINE OUT PUT.LOW K3 UREA 15 CRETENENE 3
WHAT WILL DO
ACIDATION OF URINE
ALKLAIZATION OF URINE
HEMODIALISS
C they said

408) Patient known case endocarditis will do dental procedure prophylaxis?


a) 2 g amoxicillin before procedure 1 h
b) 1 g amoxicillin after procedure
c) 2 g clindamycine before procedure 1 h
d) 1 g clindamycine after procedure
A they said>>> pt with endocarditis should have only one dose of oral amoxicillin before the dental proceadure... 2nd dose
can be given after it but only for high risk pts & usually not needed..Clindamycin given to Pts allergic to penicillin
409) Young patient on anti TB medication presented with vertigo which of the following drug cause this
a) Streptomycin
b) Ethambutol
c) Rifampicin
A they said
410) Pregnant lady 34 weeks came to you in labor. O/E, the baby is back down, transverse lie, cervix is 3 cm
dilated and bulging membrane. Her contractions are 1/ 4 minutes. U/S showed posterior fundal placentaWhat is the
management?
a) Cesarean section
b) Amniotomy
c) Oxytocin
d) Amniocentesis to assess fetal lung maturity
A they said
411) CHID AT NIGHT ESCREMING ANG YALYING IN THE MORRNIG HE DIDNT REMBERE ANY THING WHAT TO DO
REASURE THE PEARANT>>yes
412) scaniro about the pt with soical phopia trement
ssri for long time
b blockor
bihaviral therpy
A they said

413) In septic shock:


a) The mortality rate is 10 to 20%
b) Gram-negative organisms are involved exclusively
c) The majority of patients are elderly
d) The most common source of infection is alimentary tract.
e) Two or more organisms are responsible in the majority of cases
C they said>> Septic shock occurs most often in the very old and the very young. It also occurs in people who have other
illnesses.>> Diseases of the genitourinary system, biliary system, or intestinal system>> Septic shock has a high death
rate. The death rate depends on the patient's age and overall health, the cause of the infection, how many organs have
failed, and how quickly and aggressively medical therapy is started.
414) side efects of opide
musle pain
neuroginic pain
palpitation
A they said
415) PT WITH ADISSON DISSE FOR LONG TIME CAME WITH BP90/60
TACHYCARDID SWEETING. CASE OF ADDISON CRISSIS TRETMENT
GIVE HIM 0.9NACL 1L AND CRTISON ORAL
0.9.NACL WTHEN 8HOUR AND ORAL CORISION
GIVE0.9 NACL 1HOUR AND100MG IV HYDROCRTISON
0.9NACL 8HOUR AND 200MG IV HYDROCRTISON
B they said
416) transmision yersina virus>>>fela
417) tow girl tretet from anixity with AWRIi think like that aprviation what A MEANS
ALIVAIT STRESS
A.......CALM DOWN
A they said

418) Common symptoms of Hodgkin lymphoma not seen in non Hodgkin lymphoma:
a) night sweat
b) superior vena cava syndrome
c) CNS involvement
d) intussusceptions
e) bone pain
D they said
419) relative to complications that may be associated with thyroidectomy,which of the following statements is correct:
a) Tracheostomy should be performed routinely after surgical evacuation of a postoperative hematoma.
b) The clinical manifestations of postoperative hypoparathyroidism are usually evident within 24 hours.
c) A non-recurrent left anterior laryngeal nerve is present in every 100 to 200 patients.
d) When papillary carcinoma metastatizes to the lateral neck nodes, the internal jugular vein is routinely removed during
dissection
e) Inadequately treated permanent hypoparathyroidism can lead to mental deterioration.
B they said
420) Which of the following is a disease improving drug for RA
a) Aspirin
b) Steroid
c) Penicillamine
d) Hydrocloroquin
D they said
421) Elderly patient with dementia and change in his behavior (many things including agitations ) which lobe in
brain affected :
a) Frontal
b) Occipital
c) Temporal
d) Partial
e) Cerebeller

A they said
422) What is true regarding spontaneous abortion:
Can lead to infertility in future
Usually fused by any utrine abnormality
That occur on 2ed trimester..
C they said >>> can be ocured in second tremster due 2 cervical incompetence
423) Typical picture of oculomotor nerve palsy: stroke with loss of smell, which lobe is affected?
a) Frontal
b) Parital
c) Occipital
d) Temporal
D they said
424) All of the following muscles are part of rotator cuff, except:
a) Supra-spinatus.
b) Infra-spinatus.
c) Deltoid
d) Subscapularis.
e) Teres minor
C they said
425) newly diagnosed pt with pulmonary T.B started on ethambutol , streptomycin , isonized and refampcin , then he c/o
dizeness after intiating these drugs , which one should be deleted :
a/ ethambutol
b/ streptomycin
c/ isonized
d / refampcin
B they said
426) In a child with TB, all is found EXCEPT:
a) History of exposure to a TB patient.

b) Chest x-rays findings.


c) Splenomegaly.
d) Positive culture from gastric lavage.
C they said
427) pt come e rt ear pain,discharge and itching o/e erthema of ear canal e normal tympanic membrane what your ttt:
a.clean,local antibiotic and corticosteroid
b.clean and oral antibiotic
c.oral antibiotic and corticosteroid
d.clean,local antibiotic and mobbing
A they said
428) Which of the following with antipsychotic medication have rapid onset of action?
sublingual
oral
IM
IV
D they said
429) Which of the following antipsychotic associated with weight gain:
Respiridone
Quitapine
Olanzipine
Ziprasidone
C they said
430) man walking in street and saying bad words to stranger , he is not aware of his conditiond , what is the description:
flight of idea
insertion of idea
loosening of association
C they said

431) Child with fever first after 2 days he got sore throught white yellow mouth lips lesion in erythmatous
base
with gingivitis Dx?
a) HSV
b) EBV
c) CMV
d) Adenovirius
A they said
432) A 32 year-old lady work in a file clerk developed sudden onset of low back pain when she was bending on
files. Moderately severe for 3 days duration. There is no evidence of nerve root compression. What is the
proper action?
a) Bed rest for 7 to 10 days
b) Traction
c) Narcotic analgesia
d) Early activity with return to work
e) CT scan for lumbosacral vertebrae
C they said
433) A child while walking his big toes are going inward.. legs are 15 degree rotated inward.. sitting in W position with both
legs flared out behind them .. hip is 70 degree medially rotating Diagnosis:
1-Metatarsus varus
2-Metatarsus Adductus
3-Tibial torsion
4-Femoral Antiversion
4 they said
434) A wonan lactating and developed fivour and mass on her brest and a box of results to the woman chemistory
A. Cold compretion and stop brest feeding
B.hot compretion and continue breast feeding
C.other options

B they said
435) A picture with a colection of pus on the midle finger nail and a falure on augmantin for a week.
A one more week trail of augmantin
B excition under local anathesia
C.excetion under GA
Other option
B they said
436) A case of pationt with eodema in the hands chest legs .
A dry beri beri
B wet beri beri
C pelagra
Other option
B they said
437) Pt old age with osteoperosis she was advised to take ca+2 daily what amount of ca+2 she needs dsily to prevent he
osteoperosis.
A.200mg
B.400mg
C.1000mg
D.1500mg
D they said>>>> calcium requirements: 1. pre menopausal women 20-50yrs n post menopausal on oestrogen 10001200mg>>> 2. post menopausal women <65yrs n NOT on oestrogen 1500mg. 3. For men 25-65 1000mg. 4. For ALL ppl
>65yrs 1500mg>>> so we need to know more details, but in this case i'll go with D
438) Child with migrain ,the phisycian who is managing him knows he is at high risk of devoloping.
A.depression
B.hearing lose
C.scezophrenia
D.gastro oesophegal reflax disease .

A they said
439) A a child who brought with a mother that says he had a hair leasion of 22 cm the hair was cut from the edges and
rater than that unremarkable.dignosis is
A.talactomania
B.tenia capitus
C.alpecia eriata
D.other option
C they said
440) celiac disease most part affected is a-proximal part of small intestine b-distal part of small bowelc-large bowel
A they said
441) 2yrs child c\o vomiting and diarrhoa for 4days.daignosed rota virus.best test a-stool culture b-stool genaral c-stool
antigen test d-wbc
C they said
442) CANCERS MOST COMMON IN PARANASAL SINUSIS;
A/FRONTAL
B/MAXILLARY
B they said
443) 58 yrs old woman, complaint of LT eye pain and she see yellowish hallous around the lights, no loss of vision whats
the diagnosis?a-retinoblastoma b-acute glaucoma c-cataractd-diabetic retinopathy
B they said
444) 3yrs child presented after 3h of ingestion of uknown subestant c\o vomiting. salivation. lacrimation.dairrhoea.yuor
action a-gastric lavage b-atropine
c-floxetine d-characool e-epecac
B they said

445) All are complications of long term use of phenytoin, EXCEPT: a-Ataxia b- osteoporosis c- Osteomalacia d- Macrocytosis
B they said
446) medication not take by opiod depandency:
-Methadone
-Buprenorphine
-Naltrexone
-Clonidine
D they said
447) A a child who brought with a mother that says he had a hair leasion of 22 cm the hair was cut from the edges and
rater than that unremarkable.dignosis is
A.talactomania
B.tenia capitus
C.alpecia eriata
D.other option
C they said
448) table === ph7.2 pco2 40 hco3 14 ?? Respiratory acidosis, resiratorry alklosis. Metabolic acidosis. Metabolic alklosis
C they said
449) Adenosine dose should be reduced in which of the following cases:
Chronic renal failure.
Patients on thiophyline
B they said
450) With regard to tonsillectomy:
a- it is indicated in children having 2 attacks of tonsillitis a year
b- complications include development of peritonsillar abscess
c- it should be performed if parents request a prophylactic procedure
d- complications include complete dysphagia
e- post-operative secondary haemorrhage is treated with antibiotics

E they said>>>> complications of tonsillectomy include primary haemorrhage , occurring within 24 hrs and usually due to
an inadequately ligated/cauterised vessels , and secondary haemorrhage , wich is commonly due to infection and occurs
one week after procedure and treated with antibiotics.
451) regarding dermatomyositis what is true ?? 1-distal muscle weakness. 2-underlying malignancy. 3-genaralized skin rash.
>>>> not complet question demaomyosis affect skin rash plus proximal muscle weakness
452) All are complications of long term use of phenytoin, EXCEPT:a-Ataxiab- osteoporosisc- Osteomalaciad- Macrocytosis
B they said
453) Patient came with palpitation and syncope ECG showed deep S wave in lead I and V6 and tall R wave in VI :
a-RBBB
b-LBBB
c-Bifasicular block
A they said>>> The QRS duration must be more than 100 ms (incomplete block) or more than 120 ms (complete block)
There should be a terminal R wave in lead V1 (e.g. R, rR', rsR', rSR' or qR)
There should be a slurred S wave in leads I and V6.

You might also like